The question bank may take some time to load… Just enough time to stretch, blink a few times, and question your life choices — but not too long, we promise!
We recommend going Full Screen for the best experience. Have Fun !
Report a question
Endo – 2022
Questions from The 2022 Module + Annual Exam of Endocrinology
If the islet of Langerhans were a pizza, some cells would prefer the crust while others dive into the cheesy center.
1 / 110
Category:
Endo – Histology
While squinting at pancreatic histology slides like they’re trying to find Waldo, students often confuse alpha and beta cells within the islets of Langerhans. To help clarify their location, they were reminded that alpha cells are typically found in which region of the islet?
The Islets of Langerhans in the pancreas contain several endocrine cell types with a distinctive spatial arrangement (especially visible on immunostaining):
Cell Type
Hormone Secreted
Typical Location in Islet
Beta cells
Insulin
Central 🔵
Alpha cells
Glucagon
Peripheral 🔴
Delta cells
Somatostatin
Scattered throughout
Alpha cells , which produce glucagon , are typically found near the periphery of the islets.Beta cells , which secrete insulin , are mostly located centrally .
❌ Why Other Options Are Incorrect:
Only in the center of islets ❌ This describes beta cells , not alpha.
Towards the center of islets ❌ Again, this is the territory of beta cells.
Equally in both regions of islets ❌ Not true — alpha cells are not evenly distributed .
Only in the periphery of islets ❌ While alpha cells are mostly peripheral, they are not exclusively peripheral — so this option is too extreme .
Some substances can interfere with certain of thyroid hormone synthesis — even before a certain substance gets used inside the cell.
2 / 110
Category:
Endo – Physio
Which step of thyroid hormone synthesis will be affected by thiocyanate ions?
Thiocyanate ions (SCN⁻) are known to inhibit the Na⁺/I⁻ symporter (NIS) located on the basolateral membrane of thyroid follicular cells. This symporter actively transports iodide from the bloodstream into the thyroid follicular cell , a critical first step in thyroid hormone synthesis.
By competing with iodide , thiocyanate blocks its uptake , ultimately reducing thyroid hormone production (T₃ and T₄).
❌ Other Options:
Coupling → Involves linking of iodotyrosines (MIT + DIT) to form T₃/T₄; this is catalyzed by thyroid peroxidase (TPO) , not affected by thiocyanate.
Organification of thyroglobulin → Refers to iodination of tyrosine residues on thyroglobulin, also catalyzed by TPO.
Oxidation of iodine → Converts I⁻ to I₂ via TPO; not blocked by thiocyanate.
Iodination of tyrosine → The step that attaches iodine to tyrosine; also not the action site of thiocyanate.
Pancreatic islets form early, but hormone secretion ramps up by the end of the first trimester.
3 / 110
Category:
Endo – Embryology
When can glucagon and insulin be detected in the fetal plasma?
Both glucagon and insulin can be detected in fetal plasma around 11 to 13 weeks of gestation . These pancreatic hormones begin to appear after the differentiation and maturation of islet cells in the fetal pancreas.
Insulin is produced by the beta cells of the fetal pancreas and is crucial for fetal growth .
Glucagon , secreted by alpha cells , becomes functionally relevant in glucose homeostasis later in fetal life, although detectable by this stage.
This timing reflects the early endocrine function of the fetal pancreas even before the fetus becomes reliant on its own glucose metabolism.
❌ Other Options:
7–8 weeks → Too early for detectable levels; islet cells are still differentiating.
10–15 weeks → Broad range; not as precise. Detection reliably starts at ~11 weeks.
12–15 weeks → Slightly late; insulin and glucagon are already detectable at the start of this range.
15–20 weeks → By this time, both hormones are well established, not just starting to appear.
Which condition increases insulin resistance, blood pressure, and bad cholesterol—all at once?
4 / 110
Category:
Endo – Community Medicine/Behavioral Sciences
Which of these is a major cause of metabolic syndromes?
Obesity , particularly central (abdominal) obesity , is a major underlying cause of metabolic syndrome . Metabolic syndrome is a cluster of interrelated conditions—including insulin resistance, hypertension, dyslipidemia (high triglycerides and low HDL), and hyperglycemia —which significantly increase the risk for cardiovascular disease and type 2 diabetes .
Excess adipose tissue, especially visceral fat, plays a key role in promoting systemic inflammation and insulin resistance, thereby contributing to the entire spectrum of metabolic abnormalities.
❌ Why the other options are incorrect:
Hypoglycemia ➤ Not a feature of metabolic syndrome. The syndrome is instead associated with hyperglycemia or impaired glucose regulation.
Decreased appetite ➤ Opposite of the typical presentation. Most patients with metabolic syndrome may have increased appetite or overeating tendencies .
Increased sleep ➤ Poor sleep or sleep apnea may be linked to obesity, but increased sleep itself is not a cause of metabolic syndrome.
Hypotension ➤ Metabolic syndrome is associated with hypertension , not low blood pressure.
Think about what kind of capillary would allow rapid movement of small hormone molecules directly into the bloodstream—especially when timing is crucial for regulating blood glucose.
5 / 110
Category:
Endo – Histology
Which type of vessels are present between cords of the pancreas?
The pancreas, especially in its endocrine portion (the islets of Langerhans), is richly supplied by fenestrated capillaries . These vessels have small pores (fenestrations) in their endothelium which allow rapid exchange of hormones like insulin and glucagon into the bloodstream.
Fenestrated capillaries are ideally suited for endocrine organs because they permit easy passage of molecules between the blood and the glandular tissue, supporting efficient secretion and distribution of hormones .
❌ Why the other options are incorrect:
Continuous capillaries ➤ Found in places like the brain and muscles where tight regulation of exchange is required. These are not suited for hormone secretion due to the lack of pores.
Sinusoid capillaries ➤ Found in the liver, spleen, and bone marrow. These are much larger and leakier , allowing passage of cells—not typical in the pancreas.
Elastic arteries ➤ These are large vessels (like the aorta) that help manage pulsatile blood flow—not found within organs like the pancreas.
Venules ➤ Collect blood from capillaries but are not involved in active exchange; they are post-capillary vessels , not between cords of cells.
Focus on the germ layer that also contributes to most of the gut lining and digestive glands —that same layer also gives rise to the thyroid’s hormone-producing units.
6 / 110
Category:
Endo – Embryology
Which of the following gives rise to thyroid follicular cells?
Thyroid follicular cells —the cells responsible for producing thyroid hormones (T₃ and T₄)—originate from the endoderm of the primitive pharynx , specifically between the first and second pharyngeal pouches .
During embryological development, the thyroid gland begins as a median endodermal thickening in the floor of the pharynx (at the foramen cecum). It then descends in front of the pharyngeal gut to reach its final position in the neck, forming the thyroid follicles along the way.
❌ Why the other options are incorrect:
3rd pharyngeal pouch ➤ Forms the inferior parathyroid glands and thymus , not thyroid tissue.
Mesoderm ➤ Gives rise to muscles, bones, and connective tissue—not follicular cells of the thyroid.
Mesenchyme ➤ Provides supportive tissue (like connective tissue), but does not directly differentiate into hormone-secreting follicular cells.
2nd pharyngeal pouch ➤ Forms structures such as the palatine tonsils , not the thyroid gland.
Think of the organelle responsible for digesting proteins inside a cell — it breaks down thyroglobulin to liberate free hormones.
7 / 110
Category:
Endo – Physio
Which of these is mainly responsible for the exocytosis of thyroid hormone?
Thyroid hormones (T₃ and T₄) are stored in the follicular lumen bound to thyroglobulin. When needed, this complex is endocytosed into the follicular cell and then fused with lysosomes . Inside the lysosomes, proteolytic enzymes cleave T₃ and T₄ from thyroglobulin, allowing them to be released into the bloodstream via exocytosis .
❌ Why the other options are incorrect:
Peroxidase ➤ Involved in iodination and coupling reactions during synthesis of thyroid hormones, not release.
Na-K symporter ➤ Likely a misstatement of the Na⁺/I⁻ symporter (NIS) , which brings iodide into the cell — relevant to uptake, not secretion.
Thyronine ➤ Refers to the structure/base of thyroid hormones, not involved in their transport or secretion.
Pendrin ➤ A Cl⁻/I⁻ exchanger on the apical membrane, helps iodide enter the colloid — involved in hormone synthesis , not exocytosis.
What process in most cells burns ATP to push ions in opposite directions and helps set up the gradients that other transporters depend on?
8 / 110
Category:
Endo – Physio
The energy for transporting iodide across the Na-I symporter comes from which of the following?
The Na⁺/I⁻ symporter (NIS) in thyroid follicular cells actively transports iodide (I⁻) into the cells against its concentration gradient , which is essential for thyroid hormone synthesis.
This process is indirectly powered by the Na⁺/K⁺ ATPase pump , which maintains a low intracellular sodium concentration by using ATP to pump Na⁺ out and K⁺ in. The resulting sodium gradient allows sodium to flow back in via the NIS, dragging iodide along with it — a classic example of secondary active transport .
❌ Why the Other Options Are Incorrect:
Na-K-Cl pump ❌ Found mostly in the thick ascending limb of the nephron and not involved in iodide transport.
H-K ATPase ❌ Located in gastric parietal cells, where it helps secrete acid — unrelated to iodide uptake.
SGLT-2 ❌ A sodium-glucose transporter in the kidney proximal tubule, not involved in iodide transport.
GLUT-3 ❌ A glucose transporter in neurons — it does facilitated diffusion of glucose, not iodide or sodium.
Think about which blood test would still provide useful information even if the patient hasn’t fasted—and can reveal patterns over weeks, not just minutes.
9 / 110
Category:
Endo – Biochemistry
Which of the following tests will be used to measure the average blood glucose levels of a 30-year-old man over the past 3 months?
The HbA1c test (also called glycated hemoglobin or A1c) reflects the average blood glucose levels over the past 2–3 months . This is because glucose molecules slowly attach to hemoglobin in red blood cells. Since red blood cells live for about 120 days, the HbA1c gives a long-term picture of glucose control.
It’s a key diagnostic and monitoring tool for diabetes mellitus and is more stable than a single fasting or random glucose measurement.
❌ Why the Other Options Are Incorrect:
Random blood glucose test ❌ Measures sugar levels at a single moment, not over a period of time.
Hemoglobin ❌ Measures red blood cell count or oxygen-carrying capacity, not glucose control.
Fasting blood sugar test ❌ Measures blood glucose after 8 hours of fasting. Only reflects glucose at one point in time.
Glucose tolerance test ❌ Used to assess how the body handles a glucose load, mostly for diagnosing diabetes or gestational diabetes—not for long-term glucose levels.
When the body gets what it needs from the outside, does it still bother making it inside? Think about how the system reacts to an excess of a hormone mimic.
10 / 110
Category:
Endo – Pharmacology
If a person is given steroidal anti-inflammatory drugs, what will be the drug’s effect on this person?
Steroidal anti-inflammatory drugs (like prednisone or dexamethasone) mimic cortisol , the body’s natural glucocorticoid. When you give these drugs, the body senses high glucocorticoid levels through negative feedback.
As a result, the hypothalamus and anterior pituitary reduce their secretion of:
This feedback inhibition aims to prevent overproduction of endogenous cortisol.
Because the drug is acting like cortisol , the body suppresses its own hormone production—low ACTH and low CRH .
❌ Why the Other Options Are Incorrect:
High cortisol, high CRH ❌ CRH would be low due to negative feedback from high cortisol levels.
High cortisol, low ACTH ❌ Only partially correct. Cortisol is high (from the drug), ACTH is low—but CRH would also be low, so this option is incomplete.
Low cortisol, high ACTH ❌ Steroids cause high cortisol effect, not low. Also, ACTH would be suppressed.
High CRH, high ACTH ❌ Opposite of what happens—both would be suppressed by the steroid’s cortisol-like effect.
This hormone kicks in when you’re in danger—literally and metabolically. It helps mobilize energy quickly in a crisis, including when your blood sugar drops dangerously low.
11 / 110
Category:
Endo – Physio
Which hormone functions as the second line of defence to prevent hypoglycemia?
Hormonal Defense Against Hypoglycemia
The body has a well-organized hormonal hierarchy to protect against hypoglycemia (low blood glucose). These hormones are released in stages depending on the severity and duration of the glucose drop.
🥇 First Line of Defense:
🥈 Second Line of Defense:
Thus, epinephrine backs up glucagon when hypoglycemia persists or worsens.
❌ Why the Other Options Are Incorrect:
Cortisol ❌ It plays a third-line, slower role in sustaining blood glucose through gluconeogenesis and protein catabolism , but it’s not fast-acting like epinephrine.
Growth Hormone ❌ Also a late-acting hormone , it helps by reducing glucose uptake in tissues and promoting lipolysis , but it’s not a rapid responder.
Insulin ❌ Insulin lowers blood glucose. It is counter-regulatory to glucagon and epinephrine , so it’s suppressed during hypoglycemia.
Glucagon ❌ This is actually the first line of defense, not the second.
Think about how corticosteroids are used in medicine to reduce swelling, pain, and immune overactivity. What cellular organelle and vascular change might they target to reduce damage from inflammation?
12 / 110
Category:
Endo – Physio
How does cortisol exert its anti-inflammatory effects?
Mechanism of Cortisol’s Anti-Inflammatory Effect
Cortisol, a glucocorticoid hormone secreted by the adrenal cortex, plays a major role in suppressing inflammation. It achieves this effect through multiple coordinated actions on immune and vascular systems.
🔹 Key Anti-Inflammatory Mechanisms of Cortisol:
Stabilizes lysosomal membranes : This prevents the release of powerful enzymes from damaged white blood cells that could worsen tissue injury.
Reduces capillary permeability : By decreasing histamine release and vascular permeability, cortisol helps reduce swelling and edema in inflamed tissues.
Suppresses the production of pro-inflammatory cytokines like IL-1, IL-6, and TNF-α.
Inhibits immune cell proliferation , especially T-lymphocytes, and limits antibody production.
Thus, cortisol’s action is not about enhancing immunity , but rather controlling and dampening the immune response to prevent excessive tissue damage.
❌ Why the Other Options Are Incorrect:
“By increasing the phagocytosis of the damaged cells” ❌ Cortisol actually suppresses phagocytic activity of immune cells in high doses.
“By increasing the release of IL-1 from WBCs” ❌ Cortisol inhibits IL-1, a pro-inflammatory cytokine. This suppression is key to its anti-inflammatory action.
“By increasing lymphocyte production” ❌ Cortisol causes lymphopenia (reduction in lymphocyte count) by redistributing and reducing production.
“All of these” ❌ Since most of the options are wrong, this cannot be correct.
What natural daily rhythm helps restore hormonal balance by ensuring the rise and fall of cortisol follows a predictable pattern?
13 / 110
Category:
Endo – Physio
Which of the following counteracts the negative effects of cortisol?
Cortisol is a glucocorticoid hormone secreted by the adrenal cortex in response to stress and regulated by the hypothalamic-pituitary-adrenal (HPA) axis. While cortisol is essential for metabolic, immune, and stress-related functions, chronic elevation can lead to negative effects such as:
Hyperglycemia
Muscle wasting
Immunosuppression
Mood disturbances
The sleep-wake cycle (also known as the circadian rhythm ) plays a key role in modulating and resetting cortisol secretion to its physiological levels. Normally:
Cortisol peaks early in the morning (around 6–8 AM)
It declines throughout the day
It is lowest at night during deep sleep
A regular sleep-wake cycle helps prevent prolonged cortisol elevation , supports HPA axis balance , and counteracts the harmful effects of cortisol on metabolism, mood, and immunity.
❌ Why the Other Options Are Incorrect:
Stress ❌ Increases cortisol levels further — it worsens the negative effects, not counteracts them.
Hyperglycemia ❌ Is a result of excess cortisol, not a counteraction. It reflects cortisol’s action on glucose metabolism.
CRH (Corticotropin-releasing hormone) ❌ Comes from the hypothalamus and stimulates ACTH release , which then stimulates more cortisol production — it promotes, not reduces, cortisol effects.
ACTH (Adrenocorticotropic hormone) ❌ Stimulates the adrenal cortex to release cortisol — again, it increases , rather than reduces cortisol.
Which steroid would you choose if you needed the maximum anti-inflammatory effect with minimal fluid retention ?
14 / 110
Category:
Endo – Pharmacology
Which of the following is the most potent corticosteroid?
Corticosteroids vary in potency , duration , and mineralocorticoid activity . Among them, dexamethasone stands out as the most potent glucocorticoid , used especially when a strong anti-inflammatory or immunosuppressive effect is required.
Let’s compare a few key corticosteroids by their glucocorticoid potency :
Steroid
Relative Glucocorticoid Potency
Mineralocorticoid Activity
Cortisol (Hydrocortisone)
1
1
Cortisone
0.8
0.8
Prednisone
4
0.8
Dexamethasone
25–30
~0
Corticosterone
0.1
0.4
So, dexamethasone is about 25 times more potent than cortisol , and has negligible mineralocorticoid effect , making it ideal in conditions where fluid retention is undesirable (e.g. cerebral edema, brain tumors, etc.).
❌ Why the Other Options Are Incorrect:
Cortisol ❌ It’s the natural glucocorticoid but has the lowest potency on this list.
Prednisone ❌ Stronger than cortisol but still much weaker than dexamethasone .
Corticosterone ❌ Has minimal glucocorticoid potency , more involved in mineralocorticoid pathways.
Cortisone ❌ It is biologically inactive until converted to cortisol in the liver.
If the engine of the body starts running slower than usual, what symptom might reflect a reduced overall level of activity and alertness?
15 / 110
Category:
Endo – Physio
Which of the following may present in a female with hypothyroidism?
Hypothyroidism is a condition characterized by decreased production of thyroid hormones (T3 and T4) , leading to a slowed metabolism . This slowing affects multiple organ systems, and patients—especially women, who are more commonly affected—often present with:
Fatigue and somnolence (excessive daytime sleepiness)
Weight gain (despite reduced appetite)
Cold intolerance
Bradycardia
Constipation
Dry skin, hair thinning, menstrual irregularities , etc.
So, somnolence fits this picture perfectly and is a classic symptom of the generalized “slowing down” seen in hypothyroidism.
❌ Why the Other Options Are Incorrect:
Weight loss ❌ Seen in hyperthyroidism due to increased basal metabolic rate.
Goiter ❌ While a goiter can be present in hypothyroidism , especially in iodine deficiency or Hashimoto’s thyroiditis , it is not a universal symptom . The question asks what may present , so this is borderline but less characteristic than somnolence.
Tachycardia ❌ Common in hyperthyroidism , not hypothyroidism. Hypothyroid patients typically have bradycardia .
Heat intolerance ❌ Another feature of hyperthyroidism , not hypo. Hypothyroid patients are intolerant to cold .
What’s a safe long-term blood sugar control target that balances between avoiding complications of diabetes and minimizing the risk of dangerously low sugar levels?
16 / 110
Category:
Endo – Community Medicine/Behavioral Sciences
A 50-year-old diabetic man presents to the outpatient department. The medication prescribed to him will lower his Hb1Ac down to which value?
In the management of type 2 diabetes mellitus , HbA1c (glycated hemoglobin) is used to assess long-term glycemic control. The goal of therapy is not to bring the HbA1c down to completely normal levels (like 5–6%), as that may increase the risk of hypoglycemia , especially in older adults or those with comorbidities.
The American Diabetes Association (ADA) generally recommends an HbA1c target of < 7% for most non-pregnant adults, but a level slightly above 6.5% is often acceptable and safe , depending on the patient’s age, comorbidities, and risk of hypoglycemia.
Therefore, the most accurate and practical answer is:HbA1c is lowered to just above 6.5% with effective treatment—not too tight, not too loose.
❌ Why the Other Options Are Incorrect:
> 7% : This is higher than the usual therapeutic goal. While sometimes tolerated in specific populations, it’s not the primary target.
> 6% : Too strict—may risk hypoglycemia, especially in older patients.
> 10% : Far too high; indicates poor control and high risk for complications.
> 5% : Unnecessary and often unsafe to aim this low in diabetics on medication.
Which stage of development is centered around the part of the body that infants use most to explore their environment—especially when it comes to comfort and curiosity?
17 / 110
Category:
Endo – Community Medicine/Behavioral Sciences
A 5-year-old boy is frequently biting himself and putting objects in his mouth. Which psychosexual stage of development may he be experiencing?
Freud’s psychosexual stages of development describe how personality develops through a series of childhood stages where the pleasure-seeking energies of the id focus on different areas of the body.
The oral stage is the first stage , typically from birth to around 1.5 years , where the mouth is the primary source of pleasure and interaction. Behaviors such as biting , sucking , and putting objects in the mouth are natural at that age.
However, if these behaviors persist beyond the expected age , as in this 5-year-old boy, it may suggest a regression to or fixation at the oral stage, especially under stress or developmental issues.
❌ Why the Other Options Are Incorrect:
Anal stage : Focused on control and toilet training (around 1.5–3 years); fixation leads to orderliness or messiness, not oral behaviors.
Latency stage : Occurs from around 6 years to puberty; sexual impulses are suppressed, and focus is on social and intellectual development.
Genital stage : Begins at puberty; marked by mature sexual interests.
Phallic stage : Around ages 3–6; centers on the genitals and identity formation (Oedipus/Electra complex), not oral fixation.
Which early developmental stage, centered around learning control and discipline, might lead to perfectionism and compulsive behavior if disrupted?
18 / 110
Category:
Endo – Community Medicine/Behavioral Sciences
A man is very self-conscious about hygiene so he is always washing his hands and wearing gloves. Which psychosexual stage of development did he potentially skip?
According to Sigmund Freud’s psychosexual theory of development , the anal stage (typically from 1.5 to 3 years of age ) is the period during which children learn control over bodily functions , particularly toilet training . This stage is crucial for developing a healthy attitude toward order, control, and cleanliness .
If fixation or conflict occurs during this stage—such as overly strict or lenient toilet training—it may result in what Freud called:
Anal-retentive personality : Obsessive cleanliness, orderliness, and control
Anal-expulsive personality : Messiness, disorganization, and rebelliousness
The behavior described in the question—excessive cleanliness and handwashing —is typical of anal-retentive traits , suggesting unresolved issues during the anal stage .
❌ Why the Other Options Are Incorrect:
Genital stage : Occurs during adolescence; focused on mature adult sexuality and relationships. Not linked with obsessive hygiene.
Oral stage : In infancy; issues here relate more to dependency, oral fixations (e.g., smoking, nail-biting).
Phallic stage : Involves identity formation and the Oedipus/Electra complex; not directly tied to cleanliness.
Latency stage : A period of relative calm where psychosexual development is dormant; major focus is on social and intellectual development.
Consider the only thyroid condition where the immune system affects not only the gland but also tissues behind the eyes, leading to a very striking facial appearance.
19 / 110
Category:
Endo – Pathology
A 50-year-old lady presents to the emergency department with an enlarged thyroid, protruded eyes, severe palpitations, sweating, and complaints of fatigue. What is the diagnosis?
The clinical picture described is classic for Graves disease , the most common cause of hyperthyroidism , particularly in middle-aged women.
🔍 Key Clinical Features in This Case:
Enlarged thyroid → indicates goiter
Protruded eyes (exophthalmos) → hallmark of Graves disease due to autoimmune orbital fibroblast stimulation
Severe palpitations and sweating → signs of increased sympathetic activity
Fatigue → paradoxically seen in hyperthyroidism due to muscle catabolism and increased metabolic rate
🦠 Pathophysiology:
Graves disease is an autoimmune disorder where TSI (thyroid-stimulating immunoglobulins) mimic TSH and stimulate the thyroid gland excessively, leading to overproduction of T3 and T4 .
❌ Why the Other Options Are Incorrect:
Hashimoto’s thyroiditis : Causes hypothyroidism , not hyperthyroidism; commonly presents with fatigue, weight gain, and cold intolerance .
Multinodular goiter : Can lead to hyperthyroidism (toxic variety), but does not cause exophthalmos .
Myxedema : Severe hypothyroidism with features like puffy face, bradycardia, and dry skin , not palpitations and sweating.
Endemic goiter : Related to iodine deficiency ; presents with thyroid enlargement , but no exophthalmos or classic hyperthyroid signs .
Which volatile substance, commonly used in nail polish remover, can also make its way into your breath when your body burns fat instead of sugar?
20 / 110
Category:
Endo – Pathology
What is the name of the sweet odor breath that is often associated with uncontrolled diabetes?
In uncontrolled diabetes mellitus , especially Type 1 diabetes , the body cannot utilize glucose properly due to a lack of insulin. As a result, it switches to fat metabolism for energy. This process leads to the production of ketone bodies , including:
Acetone is exhaled via the lungs and gives a characteristic fruity or sweet odor to the breath. This symptom is particularly associated with diabetic ketoacidosis (DKA) —a life-threatening complication of uncontrolled diabetes.
❌ Why the Other Options Are Incorrect:
Methane breath : Associated with bacterial overgrowth or digestive disorders , not diabetes.
Halitosis : A general term for bad breath , without specifying cause—could be dental, GI, or respiratory in origin.
Garlic breath : Caused by consumption of garlic or related sulfur-containing compounds, not linked to diabetes .
Ammonia breath : Seen in kidney failure due to urea breakdown, not in diabetic ketoacidosis .
Which substance in this list has an inhibitory role in the brain—and does the same to glucagon-producing cells?
21 / 110
Category:
Endo – Physio
Which of the following substances is not a known stimulator for glucagon release?
Glucagon is a catabolic hormone secreted by the alpha cells of the pancreas. It raises blood glucose levels by stimulating glycogenolysis , gluconeogenesis , and lipolysis —primarily in the liver .
Its secretion is tightly regulated based on the body’s metabolic state , especially glucose levels. Certain substances stimulate glucagon release, while others inhibit it.
🔬 Known Stimulators of Glucagon Secretion:
Growth Hormone – indirectly enhances glucagon to mobilize energy stores.
Cortisol – a stress hormone that promotes catabolic effects including glucagon stimulation.
Cholecystokinin (CCK) – stimulates pancreatic secretion and may mildly promote glucagon release.
Theophylline – a phosphodiesterase inhibitor that increases cAMP, enhancing glucagon release.
❌ Why the Other Options Are Incorrect:
Growth hormone ✅ → Stimulates lipolysis and increases glucose levels; glucagon supports this process.
Cortisol ✅ → Increases hepatic gluconeogenesis and works synergistically with glucagon.
CCK ✅ → Released in response to fat and protein in the gut; may indirectly aid glucagon release.
Theophylline ✅ → Increases intracellular cAMP, which promotes glucagon secretion.
🚫 Correct: Gamma-butyric acid (GABA)
GABA is a central inhibitory neurotransmitter , and in the pancreatic islets , it’s actually secreted by beta cells and inhibits glucagon secretion . It does this by activating GABA-A receptors on alpha cells, leading to hyperpolarization and suppression of glucagon release.
This gland is so tiny that you could almost mistake it for a lentil seed—but it holds the key to calcium regulation.
22 / 110
Category:
Endo – Anatomy
What is the length of the parathyroid gland?
The parathyroid glands are small endocrine glands usually found posterior to the thyroid gland , often embedded in its capsule. Most individuals have four parathyroid glands (two superior, two inferior), though this number can vary.
📏 Size of Parathyroid Glands:
Despite their small size, these glands play a crucial role in calcium homeostasis , mainly by secreting parathyroid hormone (PTH) .
❌ Why the Other Options Are Incorrect:
26 mm (2.6 cm) : Far too large; this is over four times their normal length.
15 cm : Unreasonably long for any gland in the neck.
6 cm / 26 cm : These are completely inaccurate and grossly exaggerated lengths.
Which nerve, looping back from the chest, controls your vocal cords—and is dangerously close to the back of the thyroid?
23 / 110
Category:
Endo – Anatomy
Which of the following nerve is damaged during thyroidectomy?
During thyroidectomy , one of the most feared complications is injury to the recurrent laryngeal nerve , a branch of the vagus nerve (CN X). This nerve:
Innervates all intrinsic muscles of the larynx (except cricothyroid)
Controls vocal cord movement
Runs very close to the inferior thyroid artery and posterior surface of the thyroid gland
💬 Consequences of Injury:
Unilateral damage → hoarseness or weak voice
Bilateral damage → airway obstruction due to paralysis of both vocal cords , possibly life-threatening
❌ Why the Other Options Are Incorrect:
Superior laryngeal nerve : Can also be injured (especially external branch ), leading to loss of pitch modulation due to cricothyroid paralysis—but less commonly than the recurrent laryngeal nerve.
Superior thyroid artery : It’s a blood vessel, not a nerve—though it may be ligated during surgery.
Inferior thyroid artery : Also a vessel , but closely associated with the recurrent laryngeal nerve—care is taken during ligation to avoid nerve injury.
Inferior laryngeal nerve : This is actually another name for the terminal branch of the recurrent laryngeal nerve —but “recurrent laryngeal nerve” is the proper term commonly used in this surgical context.
In a patient with suspected hormone deficiency and dangerously low pressure, what should you restore before trying to name the disease?
24 / 110
Category:
Endo – Pathology
A 45-year-old man presents to the emergency department with a 2-week history of fatigue, weakness, and weight loss. He also complains of nausea, vomiting, and dizziness. He has a history of chronic corticosteroid use for rheumatoid arthritis. On examination, he is hypotensive and has hyperpigmentation of the skin. His laboratory results show low sodium, glucose, and high potassium levels. Serum cortisol is low. What is the next best step in management?
This patient is presenting with acute adrenal insufficiency (also known as adrenal crisis ), a life-threatening emergency . The symptoms and lab findings are classic:
🔍 Key Clinical Clues:
History of chronic corticosteroid use → risk of adrenal suppression
Fatigue, weight loss, hypotension, nausea, vomiting → signs of adrenal crisis
Hyperpigmentation → suggests elevated ACTH (primary insufficiency may be unmasked)
Labs :
↓ Sodium
↑ Potassium
↓ Glucose
↓ Cortisol
This triad is classic for adrenal insufficiency , especially primary , though it may be mixed with secondary if steroid withdrawal is involved.
🩺 Immediate priority is not diagnosis, but survival .
➡️ Start empiric treatment immediately :
Once stabilized, you can pursue confirmatory diagnostic tests later (e.g., ACTH level, ACTH stimulation test, CT scan, etc.).
❌ Why the Other Options Are Incorrect Now:
Morning ACTH levels : Important for diagnosis later , but delay in treatment risks death.
Dexamethasone suppression test : For Cushing’s diagnosis, not useful here.
Saline challenge test : Not a diagnostic step for adrenal insufficiency.
Abdominal CT scan : Can help detect adrenal pathology but is not urgent in acute crisis.
If a hormone is causing your blood pressure to stay high despite treatment, where should you look to find the source?
25 / 110
Category:
Endo – Pathology
A 55-year-old female presents to her primary care physician with uncontrolled hypertension. She reports no symptoms but her blood pressure is found to be consistently elevated in her recent visits. She has been on multiple anti-hypertensive medications without success. Laboratory tests reveal low plasma renin activity and high aldosterone levels. What is the next best investigation?
This patient’s uncontrolled hypertension , despite multiple medications, along with high aldosterone and low plasma renin activity (PRA) , is highly suggestive of primary hyperaldosteronism —most commonly caused by:
Once a biochemical diagnosis (↑ aldosterone + ↓ renin) is made, the next step is to localize the source of excess aldosterone production .
🖥️ Best next test :
➡️ CT scan of the abdomen , focused on the adrenal glands .
This will help detect:
❌ Why the Other Options Are Incorrect:
Renal biopsy : Not indicated here. There’s no evidence of glomerular disease or nephritic/nephrotic syndrome.
Plasma metanephrines : Used to evaluate for pheochromocytoma , which would present with episodic hypertension, palpitations, headaches, and sweating , not isolated high aldosterone.
Doppler ultrasound of abdomen : Good for renal artery stenosis , which causes secondary hyperaldosteronism (both aldosterone and renin elevated ), not what we see here.
Brain MRI : Unrelated unless neurological symptoms are present or you’re investigating secondary hypertension from central causes , which this case doesn’t suggest.
Which condition slows down every system in the body—including heart rate, reflexes, and metabolism—and often hides behind vague symptoms like fatigue and weight gain?
26 / 110
Category:
Endo – Pathology
A 30-year-old female patient, obese, nondiabetic, presented with significant weight gain, easy fatiguability, and lethargy. On examination, she looked pale, obese, and slightly confused. Her blood pressure is 170/90 mmHg, and her pulse rate is 52 beats/min. She was unable to stand from a sitting position without support and her deep tendon reflexes were diminished. With the given scenario, what is the probable diagnosis?
This 30-year-old obese female presents with a constellation of symptoms that point strongly toward hypothyroidism :
🔍 Key Clinical Features:
Weight gain & lethargy → Common in hypothyroidism due to slowed metabolism
Pallor & confusion → Suggest reduced metabolic activity and possibly anemia
Bradycardia (52 bpm) → Classic sign of reduced thyroid hormone activity on the heart
Hypertension (170/90 mmHg) → Due to increased peripheral vascular resistance in hypothyroidism
Inability to rise from sitting → Suggests proximal muscle weakness or myopathy
Diminished deep tendon reflexes (especially delayed relaxation) → Very characteristic of hypothyroidism
🧪 What labs would support this?
↓ Free T₄ , ↑ TSH (in primary hypothyroidism)
Possible normocytic or macrocytic anemia
Hyperlipidemia may also be present
❌ Why the Other Options Are Incorrect:
Cushing syndrome : Causes weight gain and hypertension, but typically features central obesity, purple striae, facial rounding (moon face) , and muscle weakness without bradycardia or delayed reflexes.
Obstructive sleep apnea (OSA) : May cause fatigue and hypertension but not bradycardia, reflex changes, or muscle weakness .
Polycystic ovarian disease (PCOS) : Usually presents with menstrual irregularity, hirsutism, acne , and infertility —not bradycardia or delayed reflexes.
Obesity : May explain weight gain and fatigue but does not explain bradycardia, reflex changes, or confusion .
Which waist size in inches marks the red line for men, after which fat around the belly starts increasing your risk of serious chronic illnesses?
27 / 110
Category:
Endo – Community Medicine/Behavioral Sciences
A 50-year-old male known case of diabetes was advised by the physician to reduce his waist circumference. Which of the following options contains a maximum cut-off for males after which the risk of comorbidity increases?
Waist circumference is a practical indicator of central (abdominal) obesity , which is a key risk factor for:
Type 2 diabetes
Cardiovascular disease
Metabolic syndrome
According to international guidelines (e.g., WHO , NCEP ATP III , and IDF ), the waist circumference cut-off for increased health risk in males is:
➡️ Greater than 102 cm (40 inches)
At or above this threshold, the risk of insulin resistance, hypertension, dyslipidemia , and other comorbidities rises significantly.
❌ Why the Other Options Are Incorrect:
Greater than 130 cm (51 inches) : Extremely high; well beyond the risk threshold
Greater than 100 cm (39 inches) : Close, but official cut-off is 102 cm
Greater than 90 cm (35 inches) : Applies to Asian male populations , not general global male population
Greater than 88 cm (34.6 inches) : Cut-off for females , not males
Which gland can swell and malfunction if this mineral is missing from the diet—especially in regions where the soil lacks it?
28 / 110
Category:
Endo – Physio
Which of the following endocrine glands is heavily dependent on the trace mineral iodine?
The thyroid gland is heavily dependent on iodine , a trace mineral essential for the synthesis of its hormones:
Thyroxine (T₄)
Triiodothyronine (T₃)
Each of these hormones contains iodine atoms :
T₃ has 3 iodine atoms
T₄ has 4 iodine atoms
Iodine is taken up from the blood into thyroid follicular cells via the sodium-iodide symporter (NIS) , then incorporated into thyroglobulin during hormone synthesis. Without adequate iodine, the thyroid cannot produce sufficient hormones, leading to:
❌ Why the Other Options Are Incorrect:
Thymus : Involved in T-cell maturation; not iodine-dependent
Pineal gland : Produces melatonin; doesn’t require iodine
Pancreas : Produces insulin and glucagon; no iodine role
Liver : Involved in metabolism and hormone conversion but does not need iodine for its function
What psychological “reward” do you earn when you resolve each of Erikson’s conflicts successfully—something like “hope,” “will,” or “wisdom”?
29 / 110
Category:
Endo – Community Medicine/Behavioral Sciences
Erikson postulated that every stage is basically a defined period of age in which a particular conflict arises. The way these conflicts are dealt with will determine the end result which can be either a successful resolution or a continuation of the conflict. If the conflict is resolved, the stage is satisfied, an inner attribute will be acquired. What is this inner attribute?
Erik Erikson’s psychosocial theory of development outlines eight stages that individuals pass through from infancy to old age. Each stage involves a central conflict or crisis that must be resolved. The resolution of this conflict shapes the person’s psychological development.
If the conflict is successfully resolved , the individual gains a positive psychological strength or “inner attribute” .
Erikson referred to this positive outcome as a “virtue” .
For example:
In the stage of Trust vs. Mistrust (infancy), successful resolution leads to the virtue of hope .
In Industry vs. Inferiority , the resulting virtue is competence .
In Identity vs. Role Confusion , the virtue gained is fidelity .
Each virtue contributes to healthy personality development and prepares the individual for challenges in later stages of life.
❌ Why the Other Options Are Incorrect:
Self-esteem : Refers to overall self-worth but is not Erikson’s specific term for the result of stage resolution.
Self-concept : General term for how one perceives themselves—not tied to stage-specific resolution.
Self-identity : Relevant to one specific stage (adolescence ) but not the universal outcome of all stages.
Self-worth : Similar to self-esteem, but again, not the term Erikson used.
Which molecules rise during chronic stress and inflammation, quietly damaging blood vessels and increasing the risk of heart attacks—especially in overweight, sleep-deprived individuals?
30 / 110
Category:
Endo – Pathology
A 35-year-old young male working in a marketing company was referred to you by a cardiologist after a minor angina attack. He complained of decreased sleep and pain in neck muscles. He works almost 10 to 12 hours a day and rides motorbikes to meet his clients in the city. He gained weight of 15 Kg in the last two months, having a body mass index of 35 Kg/m^2. What could be the possible reason for his recent attack?
This patient is a 35-year-old male with:
Among these factors, the most biochemically and clinically significant contributor to a cardiac event in such a young individual is chronic stress , which triggers low-grade systemic inflammation .
Stress activates the hypothalamic-pituitary-adrenal (HPA) axis and sympathetic nervous system , leading to the release of:
IL-6, in turn, stimulates the liver to produce C-reactive protein (CRP) , a key inflammatory marker strongly associated with:
This inflammatory state can accelerate coronary artery disease (CAD) and trigger acute coronary syndromes even in younger individuals, especially those with visceral obesity and poor sleep .
❌ Why the Other Options Are Incorrect:
Riding a motorbike : Increases musculoskeletal strain, but is not a direct cardiac risk factor .
Nature of job : Contributes to stress but is too vague ; the inflammatory response is more directly causative.
Age factor : 35 is young for typical angina; age alone is not the main issue here.
Decreased sleep : A risk factor for cardiovascular disease but mediated through inflammatory markers like IL-6 and CRP.
Which syndrome causes rapid adrenal failure due to hemorrhage, often triggered by a rapidly progressing bacterial infection in the bloodstream?
31 / 110
Category:
Endo – Pathology
A clinical study is performed involving subjects who developed Addison’s disease. They were recorded to have laboratory studies with hyponatremia, hyperkalemia, hypoglycemia, and decreased plasma cortisol. They became hypotensive. In some subjects, this disease had an acute onset in less than 2 days. Which of the following conditions is most likely to produce this acute course?
Addison’s disease refers to primary adrenal insufficiency , where the adrenal cortex fails to produce cortisol, aldosterone, and androgens . The symptoms described—hyponatremia, hyperkalemia, hypoglycemia, low cortisol, and hypotension —are classic signs of acute adrenal insufficiency .
Among the options, the only condition that causes such a sudden, life-threatening onset of Addisonian crisis is Waterhouse-Friderichsen syndrome , which involves:
Bilateral adrenal hemorrhage
Typically secondary to septicemia , especially Neisseria meningitidis
Rapid onset of hypotension, shock, DIC , and acute adrenal failure
This syndrome can develop in less than 2 days , making it the best match for the described acute course .
❌ Why the Other Options Are Incorrect:
Reactive systemic amyloidosis : Causes chronic adrenal damage; not sudden in onset.
Disseminated TB : Can cause Addison’s but typically does so gradually over time , not acutely.
Metastatic small cell carcinoma : Can invade adrenals, but the process is gradual , not explosive.
Blunt force abdominal trauma : Rarely causes bilateral adrenal hemorrhage —and even when it does, it’s not a classic cause of acute Addisonian crisis.
What condition arises when there’s no insulin to suppress fat breakdown, leading to acid build-up, dehydration, and coma—even if the blood sugar is extremely high?
32 / 110
Category:
Endo – Pathology
A 57-year-old man is found comatose. On physical examination, he has decreased skin turgor. Laboratory studies show a blood glucose of 780 mg/ dL. Urinalysis reveals ketosis and proteinuria along with 4+ glycosuria. Which of the following is the most likely diagnosis?
This 57-year-old man presents comatose with:
Severe hyperglycemia (glucose: 780 mg/dL)
Ketosis
Dehydration (evident by decreased skin turgor)
Glycosuria and proteinuria
These findings are most consistent with Diabetic Ketoacidosis (DKA) —a serious complication typically seen in Type I diabetes mellitus .
In Type I diabetes :
There’s absolute insulin deficiency due to autoimmune destruction of pancreatic beta cells .
Without insulin, glucose cannot enter cells → hyperglycemia
Fat breakdown is increased → ketone production
Ketosis leads to metabolic acidosis , dehydration, and can progress to coma
Although the patient is older than typical onset, latent autoimmune diabetes in adults (LADA) can present similarly.
❌ Why the Other Options Are Incorrect:
Cushing syndrome : Causes hyperglycemia due to cortisol excess but not ketosis or DKA .
Type II diabetes mellitus : DKA is rare in type II; more often associated with hyperosmolar hyperglycemic state (HHS) , which lacks ketosis .
Neuroendocrine tumor secreting glucagon (glucagonoma) : Can raise glucose but doesn’t cause DKA or glycosuria of this severity.
Ingestion of large quantity of sugar : May cause transient hyperglycemia and glycosuria but not ketosis or coma .
Which bone condition arises from too much parathyroid hormone eating away at bone—leaving behind cysts, hemorrhage, and pain?
33 / 110
Category:
Endo – Pathology
A 49-year-old female experienced multiple attacks of abdominal pain last year. Three times she had passed stones in the urine during these episodes of pain. Now she is experiencing pain in her right middle finger for one month. On physical examination, her finger is tender. Laboratory reports show calcium 13.7 mg/ dL, phosphate 1.9 mg/ dL, and Creatinine 1.1mg/ dL, albumin 4.8 mg/ dL. Which of the following bone lesions is she most likely to have?
This patient presents with:
These are classic features of primary hyperparathyroidism , most often caused by a parathyroid adenoma .
In this condition:
Excess parathyroid hormone (PTH) increases bone resorption , releasing calcium and phosphate into the blood.
Kidneys excrete more phosphate , leading to hypophosphatemia
Increased calcium can result in nephrolithiasis (kidney stones)
The resulting bone lesion from prolonged PTH overactivity is osteitis fibrosa cystica . It is characterized by:
Subperiosteal bone resorption
Fibrous tissue replacement in bone
Formation of brown tumors (not true neoplasms, but reactive lesions due to bone turnover and hemorrhage)
❌ Why the Other Options Are Incorrect:
Osteoma : A benign, slow-growing bone tumor; not associated with hypercalcemia or systemic symptoms.
Osteochondroma : A benign cartilage-capped bony projection; unrelated to calcium/phosphate imbalances.
Osteoid : Not a disease—refers to unmineralized bone matrix, not a lesion type.
Osteomyelitis : Bone infection, typically with fever, elevated inflammatory markers—not linked to hypercalcemia or kidney stones.
Which mineral drops dangerously if parathyroid glands are damaged—causing the nerves and muscles to become overly excitable?
34 / 110
Category:
Endo – Pathology
After undergoing total thyroidectomy for follicular carcinoma of the thyroid a 50-year-old man develops a tingling sensation and neuromuscular instability post-operatively. Which of the following laboratory test should be carried out on an urgent basis to determine the further proceedings of the therapy?
The patient is showing classic signs of hypocalcemia —tingling, neuromuscular irritability, and instability —which are common complications after total thyroidectomy . During surgery, the parathyroid glands (located on the posterior surface of the thyroid) can be:
This leads to decreased parathyroid hormone (PTH) levels, which results in hypocalcemia . Therefore, the urgent test needed is serum calcium to confirm and guide prompt calcium replacement therapy .
❌ Why the Other Options Are Incorrect:
Thyroid stimulating hormone (TSH) : Important for long-term monitoring, but not urgent post-op and does not explain neuromuscular symptoms .
Serotonin : Unrelated to calcium metabolism or thyroid surgery.
Iodine : Involved in thyroid hormone synthesis but not relevant post-thyroidectomy or for current symptoms.
Parathormone (PTH) : While helpful, PTH levels take longer to return and are not the immediate concern —serum calcium is faster and directly confirms the clinical problem.
Which enzyme breaks down stored fat—something your body doesn’t need to do when it’s already well-fed?
35 / 110
Category:
Endo – Biochemistry
Which of the following metabolic enzymes is inhibited by insulin?
Insulin is an anabolic hormone —it promotes storage of nutrients (like glucose and fat) and inhibits catabolic pathways (breakdown of stored fuels). One of the key enzymes inhibited by insulin is hormone-sensitive lipase (HSL) .
HSL breaks down triglycerides stored in adipose tissue , releasing free fatty acids into the bloodstream.
Insulin inhibits HSL to prevent unnecessary lipolysis during the fed state , when energy and nutrients are abundant.
This action helps the body store fat instead of breaking it down.
❌ Why the Other Options Are Incorrect:
Glucokinase : Stimulated by insulin; promotes glucose phosphorylation in the liver.
Glycogen synthetase : Also stimulated by insulin; promotes glycogen storage.
Phosphofructokinase : A key glycolytic enzyme that is activated by insulin.
Acetyl Co-A carboxylase : Involved in fatty acid synthesis and stimulated by insulin.
Which trace metal forms the core around which insulin molecules pack tightly together into stable hexamers inside secretory granules?
36 / 110
Category:
Endo – Biochemistry
Which of the following metal ions is required for the storage and crystallization of insulin?
Zinc ions are essential for the storage and crystallization of insulin in the secretory granules of pancreatic beta cells .
Insulin is synthesized as proinsulin , which is cleaved to form insulin + C-peptide .
In the Golgi apparatus , insulin molecules aggregate around zinc ions , forming hexamers .
These insulin-zinc hexamers are stable crystalline structures stored in vesicles until they are released by exocytosis in response to high blood glucose.
Zinc not only stabilizes insulin for storage but also plays a role in regulating its secretion and biological activity.
❌ Why the Other Options Are Incorrect:
Manganate ion : Not physiologically involved in insulin storage.
Magnesium ion : Important for ATP-dependent reactions , but not for insulin crystallization .
Potassium ion : Involved in membrane potential and insulin release , but not insulin storage .
Sodium ion : Involved in electrolyte balance , not in insulin packaging or crystallization.
Which hormone is known for pressing the “brake pedal” on nearly every digestive and endocrine process?
37 / 110
Category:
Endo – Physio
Which of the following hormones inhibits the production of cholecystokinin?
Somatostatin is a powerful inhibitory hormone secreted by the D-cells of the pancreas and gastrointestinal mucosa , as well as the hypothalamus . Its role is to suppress the secretion of several other hormones and digestive functions to slow down gastrointestinal activity .
In the GI tract , somatostatin inhibits:
By inhibiting CCK , somatostatin reduces:
This slows digestion, especially during periods of low demand.
❌ Why the Other Options Are Incorrect:
Insulin : Regulates glucose uptake; not directly involved in CCK inhibition.
Growth hormone : Stimulated by GHRH; not connected to CCK regulation.
Glucagon : Raises blood glucose but does not inhibit CCK .
Adrenaline : Modulates sympathetic responses; may reduce GI activity indirectly , but does not specifically inhibit CCK .
Why would a hormone that dissolves in water but not in fat need to stay outside the cell to signal?
38 / 110
Category:
Endo – Physio
Receptors for peptide hormones are located on the surface of cell membranes due to which of the following properties of these hormones?
Peptide hormones (like insulin, glucagon, ADH, and FSH) are hydrophilic (water-soluble) and cannot pass through the lipid bilayer of the cell membrane. Because of this:
Their receptors are located on the surface of target cell membranes.
They bind to extracellular receptors (often G-protein-coupled or tyrosine kinase receptors).
This activates second messenger systems (e.g., cAMP , IP₃ , or calcium ) inside the cell to carry out their effects.
This is in contrast to steroid and thyroid hormones , which are lipophilic and can diffuse through membranes to bind intracellular or nuclear receptors .
❌ Why the Other Options Are Incorrect:
Effector system : Refers to the mechanism activated after receptor binding , not the reason for receptor location.
Hydrophobic : Peptide hormones are not hydrophobic ; they are hydrophilic.
Lipophilic : This applies to steroid and thyroid hormones , not peptides.
High affinity for cAMP : Peptides activate receptors that produce cAMP , but their receptor location is based on membrane impermeability , not affinity for second messengers.
Which hypothalamic nucleus acts like the brain’s internal clock, adjusting your body’s rhythms based on whether your eyes see daylight?
39 / 110
Category:
Endo – Physio
The nervous pathway involving the passage of light signals from the eyes to activate or deactivate the pineal gland passes from which of the following hypothalamic nuclei?
The suprachiasmatic nucleus (SCN) of the hypothalamus acts as the body’s master circadian clock . It receives direct input from the retina via the retinohypothalamic tract in response to light .
This visual information helps the SCN regulate:
Here’s how the pathway works:
Light enters the eyes and is detected by retinal ganglion cells containing melanopsin .
Signals are sent via the retinohypothalamic tract to the SCN .
The SCN processes the light signal and communicates with the pineal gland through a multisynaptic pathway involving the paraventricular nucleus , spinal cord , and sympathetic chain .
In light , melatonin production is inhibited ; in darkness , melatonin is stimulated .
❌ Why the Other Options Are Incorrect:
Subthalamic nuclei : Involved in motor regulation , not circadian rhythms
Anterior hypothalamus : Involved in thermoregulation , not pineal control
Arcuate nuclei : Involved in appetite and hormone feedback , not light perception
Supraoptic : Produces ADH and oxytocin , not involved in circadian light signaling
When insulin is missing, which energy-producing pathway is switched on to fuel vital organs—especially when glucose can’t be used?
40 / 110
Category:
Endo – Biochemistry
Which of the following functions of glucose is activated in the absence of insulin?
In the absence of insulin , such as in fasting states or uncontrolled diabetes mellitus , the body shifts from using glucose to using fats for energy. This leads to increased lipolysis , releasing free fatty acids from adipose tissue. These fatty acids are then transported to the liver , where they undergo β-oxidation , producing acetyl-CoA , which is converted into ketone bodies .
This process is called ketogenesis , and it:
Provides an alternative energy source, especially for the brain during prolonged fasting
Can lead to ketoacidosis in insulin-deficient states like type 1 diabetes
❌ Why the Other Options Are Incorrect:
Amino acid uptake : Stimulated by insulin ; inhibited in its absence
Glycogenesis : Requires insulin ; not activated without it
Shift of potassium into cells : Insulin promotes this —in its absence, K⁺ tends to remain in ECF , possibly leading to hyperkalemia
Glucose uptake and utilization : Insulin is necessary for glucose uptake in muscle and fat via GLUT-4 ; without insulin, this process is impaired
Which second messenger, made from ATP, is known for switching on a chain reaction that helps release glucose from the liver?
41 / 110
Category:
Endo – Biochemistry
Glucagon binds to the high-affinity receptors located on the cell membranes of hepatocytes to activate which of the following for a phosphorylation cascade?
Glucagon is a peptide hormone secreted by pancreatic alpha cells in response to low blood glucose . It primarily targets hepatocytes (liver cells) to increase blood glucose levels by stimulating:
Glucagon binds to a G-protein-coupled receptor on hepatocyte membranes, which activates adenylyl cyclase . This enzyme converts ATP into cyclic AMP (cAMP) .
cAMP then acts as a second messenger , activating protein kinase A (PKA) , which initiates a phosphorylation cascade leading to increased glucose production and release.
❌ Why the Other Options Are Incorrect:
Cyclic ATP : Does not exist—ATP is the substrate, not the second messenger .
Phosphodiesterase : Breaks down cAMP; inhibits the signal, not activates it.
Inositol trisphosphate (IP₃) : Used by other hormones like ADH (via V1 receptors), not glucagon.
Cyclic GMP (cGMP) : Used in nitric oxide and ANP signaling, not involved in glucagon action .
When the body is breaking down muscle proteins to make new glucose during stress, which metabolic pathway is it relying on?
42 / 110
Category:
Endo – Biochemistry
Which of the following function do amino acids mobilized by glucocorticoids play?
Glucocorticoids (like cortisol ) are catabolic hormones that mobilize energy sources during stress and fasting. One of their key actions is on protein metabolism , where they:
These amino acids are transported to the liver , where they are used for gluconeogenesis —the production of new glucose from non-carbohydrate sources. This process helps maintain blood glucose levels , especially during fasting, stress, or prolonged illness .
❌ Why the Other Options Are Incorrect:
Ketogenesis : Involves fatty acids and occurs in the liver , not primarily driven by amino acids.
Glycolysis : Breaks down glucose for energy —amino acids are not involved in this process.
Protein synthesis : Glucocorticoids actually inhibit protein synthesis; they promote protein breakdown .
Glycogenolysis : Involves breakdown of stored glycogen , not amino acids.
Which duct is only a “side road” for bile storage and does not directly release its contents into the small intestine?
43 / 110
Category:
Endo – Anatomy
A number of structures open (related) in the 2nd part of the duodenum. Which of the following does not have an opening in the duodenum?
The 2nd part of the duodenum (descending portion) receives several important openings related to digestion. These structures drain bile and pancreatic secretions into the duodenum to aid in digestion, particularly of fats and proteins.
Structures that open into the 2nd part of the duodenum:
Pancreatic duct (main duct of Wirsung) : Joins the bile duct to form the hepatopancreatic ampulla (Ampulla of Vater) .
Bile duct (common bile duct) : Brings bile from the liver and gallbladder.
Accessory pancreatic duct (duct of Santorini) : Sometimes drains the pancreas independently into the minor duodenal papilla.
Ampulla of Vater : The common channel where the bile duct and pancreatic duct open into the major duodenal papilla .
✅ All of these open directly into the 2nd part of the duodenum .
❌ Why the Cystic Duct Is Incorrect:
The cystic duct connects the gallbladder to the common bile duct .
It does not open into the duodenum directly .
Instead, it joins the common hepatic duct to form the common bile duct , which later opens into the duodenum.
Which hormone, when chronically elevated, tells the body to make more glucose and at the same time makes muscles ignore insulin?
44 / 110
Category:
Endo – Pathology
A 45-year-old male was diagnosed with adrenal diabetes as a result of high serum levels of cortisol due to hypersecretion of the adrenocorticotrophic hormone. Which of the following is associated with the above mentioned condition?
The condition described is adrenal diabetes , which results from chronic excess cortisol , often due to ACTH hypersecretion (e.g., Cushing disease ).
Cortisol is a glucocorticoid hormone that:
Promotes gluconeogenesis in the liver
Reduces glucose uptake in skeletal muscle and adipose tissue
Leads to insulin resistance —cells don’t respond to insulin as effectively
As a result:
Blood glucose levels rise
Skeletal muscle cells become less sensitive to insulin
This can mimic or contribute to the development of type 2 diabetes
This explains why high cortisol levels can lead to hyperglycemia , termed “adrenal diabetes.”
❌ Why the Other Options Are Incorrect:
Enhanced deposition of fatty acids in adipose tissue : Cortisol actually promotes lipolysis , especially in limbs, though it causes central fat deposition .
Absence of beta cells of pancreas : Seen in type 1 diabetes , not adrenal diabetes.
Decrease in blood glucose level : Cortisol raises , not lowers, blood glucose.
Enhanced utilization of glucose by cells : The opposite occurs—glucose uptake is impaired .
What would your brain and adrenal glands do if you were constantly taking high doses of hormones that they normally try to produce?
45 / 110
Category:
Endo – Pharmacology
Which of the following results will the reports show in a 35-year-old patient taking pharmacological doses of steroidal antiinflammatory drugs for asthma?
Pharmacologic doses of steroidal anti-inflammatory drugs (such as prednisone ) provide exogenous glucocorticoids , which mimic the action of cortisol . These high levels of glucocorticoids exert negative feedback on the hypothalamic-pituitary-adrenal (HPA) axis , leading to:
↓ Corticotropin-releasing hormone (CRH) from the hypothalamus
↓ Adrenocorticotropic hormone (ACTH) from the anterior pituitary
↓ Endogenous cortisol production from the adrenal cortex
Even though cortisol is high pharmacologically , the body detects high levels and shuts down its own CRH and ACTH , resulting in low endogenous cortisol levels .
If you stop steroids abruptly, the body cannot immediately produce cortisol, leading to adrenal insufficiency .
❌ Why the Other Options Are Incorrect:
High ACTH, low cortisol : Seen in primary adrenal insufficiency (e.g., Addison’s disease), not in steroid use.
High CRH, low cortisol : Seen in secondary adrenal insufficiency due to pituitary dysfunction—not due to exogenous steroids.
Low CRH, high cortisol : Would occur early during steroid use, but cortisol here refers to endogenous levels , which would be low over time.
High CRH, high cortisol : Seen in Cushing disease (ACTH-producing tumor) , not steroid use.
When you’ve burned through your stored sugar, what process kicks in to make new sugar from scratch—especially to keep your brain running?
46 / 110
Category:
Endo – Biochemistry
In the United States, people fast for nearly 20 hours in the month of Ramadan in the summer. which one of the following metabolic processes maintains blood glucose levels during this period?
During prolonged fasting (such as 18–20 hours in Ramadan), the body’s initial glucose reserves become depleted. Here’s how the body adapts:
First 12–18 hours : Blood glucose is maintained by glycogenolysis (breakdown of stored glycogen in the liver).
Beyond 18 hours : Glycogen stores run out , and the body shifts to gluconeogenesis —the process of synthesizing glucose from non-carbohydrate sources like:
This process mainly occurs in the liver (and to a lesser extent, the kidney), and it’s essential to supply glucose to the brain and red blood cells , which rely heavily on glucose for energy.
❌ Why the Other Options Are Incorrect:
Glycogenolysis : Important early in fasting but insufficient during prolonged fasts.
Glycogen synthesis : Happens in fed state , not during fasting.
Protein synthesis : Requires energy and nutrients—inhibited during fasting .
Triglyceride synthesis : Also occurs during fed state —fasting promotes fat breakdown , not storage.
Which glucose transporter works like a glucose “sensor” in both the pancreas and liver—only becoming significantly active when blood sugar is high?
47 / 110
Category:
Endo – Physio
During hyperglycemia, blood glucose enters the beta cells of the pancreas by which of the following glucose transporter?
In pancreatic beta cells , GLUT-2 is the glucose transporter responsible for allowing glucose to enter the cell during hyperglycemia (high blood glucose levels).
GLUT-2 is a low-affinity, high-capacity transporter.
It allows glucose to freely move in and out of the cell depending on the concentration in the blood.
This makes it ideal for sensing blood glucose levels —when glucose levels are high, more glucose enters the beta cell.
Once inside, glucose is metabolized, ATP increases, K⁺ channels close, Ca²⁺ enters, and insulin is secreted .
❌ Why the Other Options Are Incorrect:
GLUT-4 : Found in muscle and adipose tissue ; insulin-dependent, not present in beta cells.
GLUT-5 : Transports fructose , not glucose; found in the small intestine and sperm .
GLUT-1 : Found in RBCs and the blood-brain barrier ; not the main transporter in beta cells.
GLUT-3 : Found in neurons ; has high affinity for glucose, but not present in pancreatic beta cells.
What hormone released by the intestine enhances insulin secretion in response to a meal—but not when glucose is simply injected into a vein?
48 / 110
Category:
Endo – Physio
In a laboratory experiment, it was observed that oral glucose administration results in a much larger increase in serum insulin levels than intravenous glucose administration. Which one of the following hormones causes this effect?
The observation that oral glucose triggers a greater insulin response than the same amount of glucose given intravenously is known as the “incretin effect.” This effect is primarily mediated by incretin hormones , the most important of which is:
➡️ Gastric dependent insulinotropic peptide (GIP) (also known simply as glucose-dependent insulinotropic polypeptide )
GIP is secreted by K-cells in the duodenum and jejunum in response to oral nutrient intake—especially glucose and fat .
It stimulates insulin release from pancreatic beta cells , but only when glucose levels are elevated —making it glucose-dependent .
This is why oral glucose elicits a stronger insulin release than IV glucose—the gut hormones (like GIP) amplify the signal.
❌ Why the Other Options Are Incorrect:
Glucagon : Increases blood glucose , not insulin; acts in opposition to insulin.
Somatostatin : Inhibits insulin, glucagon, and GIP —not responsible for increasing insulin levels.
Leptin : Regulates appetite and energy balance , not involved in acute insulin release.
Gastrin : Stimulates acid secretion in the stomach; not directly involved in insulin release.
Which protein acts like a shuttle inside intestinal cells to help move calcium safely after it’s absorbed—under the instruction of vitamin D?
49 / 110
Category:
Endo – Physio
Vitamin D acts on the intestinal epithelial cells to help form which of the following proteins to increase calcium absorption from the intestine?
Vitamin D , particularly its active form 1,25-dihydroxyvitamin D₃ (calcitriol) , enhances calcium absorption from the small intestine . It does this by:
Stimulating intestinal epithelial cells (especially in the duodenum)
Inducing the synthesis of calbindin , a calcium-binding protein
Calbindin helps:
Bind calcium inside the enterocytes
Transport it across the cell cytoplasm
Facilitate safe and efficient movement of calcium to the basolateral membrane, where it is released into the bloodstream
This is a crucial mechanism by which vitamin D maintains calcium homeostasis , especially when dietary calcium is limited.
❌ Why the Other Options Are Incorrect:
Calcineurin : A phosphatase involved in immune signaling—not in calcium absorption in the gut
Calsequestrin : Found in the sarcoplasmic reticulum of muscle cells—stores calcium in muscle, not intestines
Calcitonin : A hormone secreted by the thyroid that lowers blood calcium , not a protein made by intestinal cells
Calmodulin : A calcium-sensing protein found in many tissues, but not involved in intestinal calcium transport
Which part of the nephron is the key site where PTH fine-tunes calcium retention to give the bloodstream a quick calcium boost?
50 / 110
Category:
Endo – Physio
Which of the following is the action of parathyroid to rapidly increase serum calcium levels?
Parathyroid hormone (PTH) plays a vital role in rapidly increasing serum calcium levels , and one of its quickest effects is on the kidneys . Specifically, it:
Stimulates reabsorption of calcium in the distal convoluted tubules (DCT) of the nephron
This action prevents urinary loss of calcium, helping to raise blood calcium levels
This is part of a multi-pronged mechanism by which PTH increases calcium:
Bone resorption (longer-term effect)
Increased calcium reabsorption in the DCT
Decreased phosphate reabsorption in the proximal tubules (to avoid calcium-phosphate precipitation)
Activation of vitamin D (calcitriol) to enhance intestinal calcium absorption
❌ Why the Other Options Are Incorrect:
Calcium and phosphate absorption from the entire nephron : Incorrect—PTH has opposing effects : increases calcium reabsorption, decreases phosphate reabsorption.
Phosphate from proximal convoluted tubules : PTH actually inhibits phosphate reabsorption here, promoting its excretion .
Calcium from proximal convoluted tubules : PTH has minimal effect on calcium reabsorption in the proximal tubule; the distal tubule is the main target .
Phosphate from distal convoluted tubule : Phosphate handling occurs mainly in the proximal tubule , not the distal.
If TSH can be boosted by giving TRH, where must the original defect be—in the gland that produces TSH or the one that stimulates it?
51 / 110
Category:
Endo – Pathology
Lab investigations of a 35-year-old woman revealed low serum levels of thyroxine, and serum level of thyroid-stimulating hormones was also down but a rapid increase in TSH level was seen when given a synthetic thyroid releasing factor. Which of the following can be the reason for her hypothyroidism?
This patient has low levels of both thyroxine (T₄) and thyroid-stimulating hormone (TSH) , which suggests that the thyroid gland itself is not the primary problem . Instead, the issue lies upstream —in the hypothalamic-pituitary axis .
The key diagnostic clue here is the rapid increase in TSH after administration of synthetic thyrotropin-releasing hormone (TRH) . This indicates that:
This points to a hypothalamic origin of the hypothyroidism—a condition called tertiary hypothyroidism .
❌ Why the Other Options Are Incorrect:
Posterior pituitary : Releases ADH and oxytocin , not involved in thyroid regulation.
Thyroid receptor : A defect here would cause resistance to thyroid hormone , not low T₄ with low TSH.
Thyroid follicles : Affected in primary hypothyroidism , where TSH would be high due to lack of feedback inhibition—not the case here.
Anterior pituitary : If it were the problem (secondary hypothyroidism), it would not respond to TRH stimulation —but here it does, so it’s functioning properly.
Which hormone is produced by the pituitary and acts like a thermostat, adjusting up or down based on circulating thyroid hormone levels?
52 / 110
Category:
Endo – Pathology
A 29-year-old man complains of weight gain, constipation, decreased energy, lethargy, and dry skin for the past 6 months. He was diagnosed with hypothyroidism and was put on synthetic thyroid hormone therapy. A decrease in which of the following hormonal levels should be observed to find if the treatment is effective?
In primary hypothyroidism , the thyroid gland underproduces thyroid hormones (T₃ and T₄), which leads to elevated TSH levels due to feedback from the anterior pituitary trying to stimulate the thyroid.
When a patient is treated with synthetic thyroid hormone (usually levothyroxine, a form of T₄) :
T₄ levels rise in the blood
This feeds back to the pituitary
Resulting in a decrease in TSH levels
Therefore, TSH is the most sensitive and reliable marker for monitoring treatment response in primary hypothyroidism.
❌ Why the Other Options Are Incorrect:
Vitamin A : Unrelated to thyroid function or therapy monitoring.
Free T₄ : Although it should rise with therapy, TSH is more reliable for long-term monitoring.
Plasma iron : May be affected by metabolism, but not specific to thyroid therapy response.
Plasma cholesterol : Often elevated in hypothyroidism and may improve, but it’s indirect and variable .
Free T₃ : Less reliable and fluctuates more ; not the preferred marker for monitoring therapy.
Which specific water channel gets inserted into the facing side of the kidney tubule in response to ADH, allowing water to be pulled back into the body?
53 / 110
Category:
Endo – Physio
The antidiuretic hormone stimulates the movement of which of the following intracellular protein to the luminal side of the collecting tubules?
Antidiuretic hormone (ADH) , also known as vasopressin , plays a crucial role in water reabsorption in the collecting ducts of the kidney.
ADH binds to V2 receptors on basolateral membranes of principal cells in the collecting ducts.
This triggers a cAMP signaling cascade that leads to the movement of intracellular vesicles containing aquaporin-2 (AQP2) channels to the apical (luminal) membrane .
Once inserted, these AQP2 channels allow water reabsorption from the tubular fluid , concentrating the urine.
Other aquaporins are located on different membranes or in other tissues , but only AQP2 is regulated by ADH at the luminal side .
❌ Why the Other Options Are Incorrect:
Aquaporin-1 : Found in the proximal tubules and descending loop of Henle —not regulated by ADH .
Aquaporin-3 & Aquaporin-4 : Located on the basolateral membrane of collecting duct cells, helping water exit the cell into the interstitium—not inserted into the luminal side .
Aquaporin-5 : Found in salivary and lacrimal glands , not involved in renal water reabsorption .
When your body is under stress or fasting, which hormone helps shift the energy source away from glucose toward stored fat?
54 / 110
Category:
Endo – Physio
Which of the following statements accurately states the action of growth hormone on metabolism?
Growth hormone (GH) plays a key role in regulating metabolism , especially during fasting or stress. One of its main metabolic effects is to promote lipolysis —the breakdown of fat stores —leading to an increase in free fatty acids in the blood. These fatty acids are then used for energy , sparing glucose for use by the brain and other vital organs.
This is why GH is considered diabetogenic : it reduces glucose uptake and increases blood glucose, while promoting fat breakdown for energy.
❌ Why the Other Options Are Incorrect:
Inhibition of insulin-like growth factor : GH actually stimulates IGF-1 production, especially from the liver.
Increasing the usage of carbohydrates : GH tends to reduce glucose uptake and utilization, not increase it.
Decreasing the rate of photosynthesis : Humans don’t photosynthesize—this is irrelevant to human metabolism .
Decreasing the mobilization of fats from adipose tissue : GH does the opposite —it increases fat mobilization.
Which cells are responsible for laying down new bone in growing children and are stimulated by a hormone that GH signals the liver to release?
55 / 110
Category:
Endo – Physio
On which of the following structures does the growth hormone exert its function indirectly through IGF-1?
Growth hormone (GH) , secreted by the anterior pituitary, promotes growth and metabolism. A **major part of its growth-promoting effects is indirect , acting through Insulin-like Growth Factor 1 (IGF-1) .
❌ Why the Other Options Are Incorrect:
Liver : Produces IGF-1 in response to GH, but it’s not the site of GH’s indirect action —it’s the source of IGF-1.
Cardiac muscles : GH may influence cardiac growth/metabolism, but not primarily through IGF-1 .
Skeletal muscle : GH can act here directly and indirectly , but bone growth via epiphyseal cartilage is the classic indirect IGF-1-mediated target.
Adipose tissue : GH affects fat metabolism directly , promoting lipolysis—not primarily through IGF-1.
Which layer gives rise to the lining of the digestive and respiratory tracts—and also forms the hormone-producing cells of the thyroid?
56 / 110
Category:
Endo – Embryology
A patient comes to the outpatient department complaining of swelling in front of the neck. After examination and investigation, a diagnosis of the enlarged thyroid gland was made. Which of the following structures gives rise to the enlarged gland?
The thyroid gland originates from a midline endodermal thickening in the floor of the primitive pharynx during the 4th week of embryonic development . This site is located at the future foramen cecum on the tongue.
The thyroid descends through the neck via the thyroglossal duct , which later disappears.
It reaches its final position anterior to the trachea by the 7th week of development.
The follicular cells (which produce thyroid hormones) are derived from the endoderm of the primitive pharynx.
❌ Why the Other Options Are Incorrect:
Rathke’s pouch : Gives rise to the anterior pituitary , not the thyroid.
3rd pharyngeal pouch : Forms the inferior parathyroids and thymus , not the thyroid gland.
First pharyngeal pouch : Forms structures in the ear and auditory tube , unrelated to thyroid development.
Dorsal bud : Related to pancreatic development , not the thyroid.
If the pancreas arises from the foregut, where might stray pancreatic cells accidentally settle during development?
57 / 110
Category:
Endo – Embryology
The ectopic pancreas can be seen at which of the following locations of the body?
Ectopic (or heterotopic) pancreas refers to pancreatic tissue located outside its normal anatomical position , without any vascular or anatomical connection to the main pancreas. It arises due to abnormal migration of pancreatic tissue during embryological development .
The most common sites of ectopic pancreas are:
These are all foregut or midgut-derived structures—where the pancreas itself originates—making the gastric antrum a common and logical site.
❌ Why the Other Options Are Incorrect:
Rectum : Derived from hindgut; ectopic pancreatic tissue is rarely, if ever, found here
Colon : Also hindgut; not a common site for pancreatic heterotopia
Trachea : Part of the respiratory system , ectopic pancreas is not seen here
Esophagus : While it shares embryological origin with the foregut, ectopic pancreas in the esophagus is extremely rare
Which germ layer gives rise to the internal lining of the gut tube and its glandular outgrowths like the pancreas and liver?
58 / 110
Category:
Endo – Embryology
Which one of the following gives rise to the ventral and dorsal pancreatic buds?
The ventral and dorsal pancreatic buds arise from the endoderm of the foregut during embryonic development. These two buds eventually fuse to form the pancreas :
Dorsal pancreatic bud : Forms most of the pancreas (body, tail, part of the head)
Ventral pancreatic bud : Forms the uncinate process and inferior part of the head ; it rotates posteriorly to fuse with the dorsal bud
The endodermal origin also explains why the pancreas has both endocrine and exocrine components , similar to other foregut-derived organs (e.g., liver, gallbladder).
❌ Why the Other Options Are Incorrect:
Mesoderm : Gives rise to connective tissue, muscle, and blood vessels , not the glandular epithelium of the pancreas
Ectoderm : Forms skin, neural tissue—not involved in pancreatic development
Endoderm of the esophagus : Too specific and incorrect—the pancreas arises from endoderm of the foregut , but not directly from the esophageal region
Mesenchyme : Supports organ development (stroma), but does not form the pancreatic buds themselves
What type of blood vessel would endocrine tissues need to let small, soluble hormones into the bloodstream quickly—but not allow cells or large proteins to leak out?
59 / 110
Category:
Endo – Histology
Which one of the following is the special type of vessel found around endocrine glands?
Endocrine glands secrete hormones directly into the bloodstream , so they require a specialized blood supply that allows rapid exchange between glandular cells and blood. For this reason, the blood vessels surrounding endocrine glands are typically continuous fenestrated capillaries .
These capillaries have:
Tight endothelial linings (continuous basement membrane)
Fenestrations (pores) in the endothelial cells
High permeability to small molecules and hormones
They allow efficient uptake of secreted hormones without allowing larger molecules or cells to pass through.
Commonly found in:
❌ Why the Other Options Are Incorrect:
Venules : Drain capillary beds but are not primary sites of exchange.
Discontinuous capillaries : Found in liver, spleen, bone marrow ; allow passage of large molecules and cells—not ideal for hormone exchange.
Arterioles : Control blood flow into capillaries; not the exchange vessels themselves.
Discontinuous fenestrated capillaries : Not a recognized or physiologically accurate category—capillaries are either continuous, fenestrated, or sinusoidal (discontinuous).
Which type of gland can both digest your food and regulate your blood sugar at the same time?
60 / 110
Category:
Endo – Histology
Which one of the following classifications best fits the nature of pancreatic secretions?
The pancreas is classified as a mixed gland because it has both:
Exocrine functions (~98% of its mass):
Acinar cells secrete digestive enzymes (amylase, lipase, proteases) into ducts → pancreatic duct → duodenum
These secretions are part of the exocrine system
Endocrine functions (~2%):
Islets of Langerhans contain:
These hormones are secreted directly into the bloodstream , making this an endocrine function
Thus, the pancreas performs both exocrine and endocrine roles , making it a mixed gland .
❌ Why the Other Options Are Incorrect:
Purely endocrine : Incorrect—only a small portion (islets) is endocrine; most of the pancreas is exocrine.
Purely exocrine : Also incorrect—ignores the vital hormone-producing function of the islets.
Purely mucous : The pancreas does not produce mucous; it secretes digestive enzymes (serous).
Purely serous : The exocrine part is serous, but the gland also has endocrine components—so it is not purely serous .
Which storage structures in the posterior pituitary hold the messenger chemicals made in the hypothalamus, waiting for the signal to be released?
61 / 110
Category:
Endo – Histology
Which of the following substances is stored in the herring bodies of the pars nervosa?
Herring bodies are specialized structures found in the pars nervosa (posterior pituitary). They are dilated terminal portions of axons from hypothalamic neurons—specifically those from the paraventricular and supraoptic nuclei .
These axon terminals store and release neurosecretory material , which primarily includes the hormones :
These hormones are synthesized in the hypothalamus , transported down the axons via the hypothalamo-hypophyseal tract , and stored in Herring bodies until released into the bloodstream in response to neural stimuli.
❌ Why the Other Options Are Incorrect:
Residual material : Not specific and not what is stored in Herring bodies.
Hormones : Technically correct, but too general —hormones are part of neurosecretory material , which also includes carrier proteins like neurophysin .
Enzymes : Not stored here; enzymes are not part of the secretory product.
Amorphous material : Vague and nonspecific—Herring bodies contain structured secretory vesicles .
Which part of the pituitary, derived from Rathke’s pouch, lies in the middle and holds onto the ability to darken the skin in some species?
62 / 110
Category:
Endo – Histology
A histologist observed a slide consisting of basophilic polygonal cells called melanotrophs. Which part of the pituitary gland he must be observing?
The pars intermedia is the thin layer of the pituitary gland located between the pars distalis (anterior pituitary) and the neurohypophysis (posterior pituitary). It contains:
Basophilic polygonal cells known as melanotrophs
These cells produce melanocyte-stimulating hormone (MSH) , derived from the same precursor as ACTH
In humans, the pars intermedia is rudimentary and less prominent than in some animals, but histologically, it can still be identified by these basophilic melanotrophs , often arranged near colloid-filled cystic remnants of Rathke’s pouch.
❌ Why the Other Options Are Incorrect:
Pars distalis : Contains acidophils, basophils , and chromophobes , but not melanotrophs .
Neurohypophysis : Made up of pituicytes and unmyelinated nerve fibers ; lacks hormone-producing cells like melanotrophs.
Infundibulum : Connects the hypothalamus to the pituitary; contains nerve fibers , not melanotrophs.
Pars tuberalis : Surrounds the infundibulum; primarily contains basophilic cells , but not melanotrophs or MSH-producing cells.
Which region of the pituitary houses the colorful variety of hormone-producing cells organized around leaky blood vessels to allow rapid hormone entry into circulation?
63 / 110
Category:
Endo – Histology
A histologist observed a slide in which cells were arranged in cords around fenestrated capillaries. Some cells were acidic, others were basic, and some didn’t take up any stain. Which part of the pituitary gland he must be observing?
The description matches the pars distalis , which is the main and largest part of the anterior pituitary (also called the adenohypophysis ). It is characterized by:
Cells arranged in cords or clusters
Rich fenestrated capillary network
Three major staining cell types:
Acidophils : stain pink/red (e.g., GH, prolactin-secreting cells)
Basophils : stain blue/purple (e.g., ACTH, FSH, LH, TSH-secreting cells)
Chromophobes : appear pale; do not take up much stain
These features are distinctive for the pars distalis , which is the functional core of anterior pituitary hormone secretion .
❌ Why the Other Options Are Incorrect:
Infundibulum : Part of the posterior pituitary stalk , contains unmyelinated axons—not endocrine cells with varied staining.
Pars tuberalis : Wraps around the infundibulum; contains mostly basophilic cells, not the full variety seen in pars distalis.
Adenohypophysis : A general term for the entire anterior pituitary ; while correct in scope, it’s less specific than pars distalis.
Neurohypophysis : The posterior pituitary , made up of pituicytes and nerve fibers —lacks chromophils, and does not show acidophils/basophils .
Which tiny glands behind the thyroid constantly monitor calcium levels in the blood and respond immediately when they drop?
64 / 110
Category:
Endo – Anatomy
Which of the following structures is responsible for releasing a hormone that regulates blood levels of phosphate and calcium?
The parathyroid glands release parathyroid hormone (PTH) , which is the primary regulator of blood calcium and phosphate levels .
PTH acts to:
This fine regulation maintains calcium-phosphate balance , which is vital for muscle contraction, nerve transmission, and bone health .
❌ Why the Other Options Are Incorrect:
Thyroid gland : Produces calcitonin , which lowers calcium, but its role is minor compared to PTH .
Adrenal gland : Produces hormones like cortisol, aldosterone, and catecholamines , but not involved in calcium/phosphate balance .
Thymus : Important for immune system development (T-cell maturation), not endocrine control of minerals .
Posterior pituitary gland : Releases ADH and oxytocin , unrelated to calcium or phosphate regulation.
Which nucleus near the third ventricle plays a key role in social bonding and uterine contractions, sending its signals through long axons to the posterior pituitary?
65 / 110
Category:
Endo – Physio
Which of the following nuclei of the hypothalamus secrete oxytocin?
Oxytocin is synthesized in the hypothalamus , specifically in the:
Paraventricular nucleus (main source)
Supraoptic nucleus (produces mostly ADH, but some oxytocin too)
After synthesis, oxytocin is transported down axons to the posterior pituitary , where it is stored and released into the bloodstream. Oxytocin plays key roles in:
Milk ejection during lactation
Uterine contractions during labor
Social bonding and maternal behaviors
❌ Why the Other Options Are Incorrect:
Preoptic nucleus : Involved in thermoregulation and GnRH secretion—not oxytocin.
Suprachiasmatic nucleus : The body’s circadian pacemaker ; regulates sleep-wake cycles, not hormone release.
Arcuate nucleus : Regulates appetite and prolactin inhibition —not the source of oxytocin.
Supraoptic nucleus : While it does produce some oxytocin , it is primarily responsible for ADH synthesis. The paraventricular nucleus remains the main source for oxytocin.
Which part of the adrenal gland contains cells that function like neurons, respond to sympathetic stimulation, and release stress hormones into the blood?
66 / 110
Category:
Endo – Histology
A histology slide showed a section of tissue containing chromaffin cells with abundant fairly electron-dense and moderately electron-dense granules. The tissue corresponds to what section of the adrenal gland?
Chromaffin cells are a key feature of the adrenal medulla , the inner part of the adrenal gland. These cells are modified postganglionic sympathetic neurons that secrete catecholamines —epinephrine and norepinephrine —in response to stress.
On histology slides (especially under electron microscopy ), chromaffin cells appear with:
Abundant granules : which are electron-dense (especially norepinephrine granules) or moderately electron-dense (epinephrine granules).
A highly vascularized background, reflecting the gland’s endocrine function.
These features distinguish the medulla from the adrenal cortex , which is made up of three layers (zona glomerulosa, fasciculata, and reticularis) that secrete steroid hormones , not catecholamines.
❌ Why the Other Options Are Incorrect:
Zona glomerulosa : Secretes aldosterone ; contains columnar cells arranged in clusters, not chromaffin cells .
Zona fasciculata : Secretes cortisol ; cells appear vacuolated due to lipid droplets, not granulated like chromaffin cells.
Zona reticularis : Produces androgens ; has smaller cells with a darker cytoplasm, lacks chromaffin cells .
None of these : Incorrect—medulla is clearly the correct answer.
If the thyroid gland isn’t responding properly, how would the pituitary gland try to correct the hormone imbalance?
67 / 110
Category:
Endo – Pathology
A person is diagnosed with hypothyroidism and goiter. Her laboratory tests most likely show which of the following?
In primary hypothyroidism , the thyroid gland fails to produce enough thyroid hormones (T₃ and T₄) . As a result, the pituitary gland compensates by increasing secretion of thyroid-stimulating hormone (TSH) to stimulate the thyroid.
This combination—low T₃/T₄ and high TSH —is characteristic of primary hypothyroidism , and if prolonged, the persistent stimulation by TSH leads to goiter formation (thyroid gland enlargement).
❌ Why the Other Options Are Incorrect:
Decreased TSH and decreased T₃, T₄ : Suggests secondary hypothyroidism (problem in the pituitary), not the most common form associated with goiter.
Increased TSH and increased T₃, T₄ : Would be seen in TSH-secreting pituitary adenoma , not hypothyroidism.
Decreased TSH and increased T₃, T₄ : Seen in hyperthyroidism , the opposite condition.
None of these : Incorrect—there is a clearly correct pattern here.
What kind of transport allows a negatively charged ion to enter a cell against its gradient, using the movement of another ion down its gradient, without using ATP directly?
68 / 110
Category:
Endo – Physio
Iodine transport in follicular cells of the thyroid gland is an example of which type of transport?
Iodine transport into thyroid follicular cells is carried out by the sodium-iodide symporter (NIS) . This transporter moves iodide (I⁻) into the cell against its concentration gradient by coupling it with sodium (Na⁺) moving down its own gradient .
This is a classic case of secondary active transport :
So, iodide transport is indirectly dependent on ATP , hence it’s termed secondary .
❌ Why the Other Options Are Incorrect:
Simple diffusion : Iodide is charged and cannot pass through the lipid membrane without a transporter.
Primary active transport : Involves direct use of ATP , such as the Na⁺/K⁺ pump—not the case here.
None of these : Incorrect—there is a well-defined mechanism (secondary active transport).
Facilitated diffusion : Passive process; moves substances down their gradient—not against , as in this case.
Damage to which area of the hypothalamus would likely lead to decreased interest in food and water—suggesting it normally acts to keep us nourished and hydrated?
69 / 110
Category:
Endo – Physio
A 30-year-old patient presents with anorexia after suffering from a head trauma one week ago. Which hypothalamic nucleus is involved in the regulation of thirst and hunger?
The lateral nucleus of the hypothalamus plays a key role in stimulating hunger and thirst . It is often referred to as the “feeding center ” of the brain. When this area is damaged , as can occur in head trauma , it can lead to loss of appetite (anorexia) and decreased fluid intake .
Stimulation of the lateral nucleus promotes feeding behavior, while lesions here result in aphagia (refusal to eat) and weight loss .
❌ Why the Other Options Are Incorrect:
Posterior nucleus : Primarily involved in heat conservation and sympathetic responses , not hunger or thirst.
Ventral nucleus : Likely a confusion with the ventromedial nucleus , which actually inhibits hunger (satiety center), but not the primary answer in this scenario.
None of these : Incorrect—the lateral nucleus is clearly involved .
Anterior nucleus : Involved in temperature regulation (cooling) and parasympathetic activity , not feeding behavior.
Which hormone acts almost like a reflex response—released from the brain in response to suckling—and triggers smooth muscle contraction, not milk creation?
70 / 110
Category:
Endo – Physio
Which hormone is involved in the ejection of milk from mammary glands during lactation?
Oxytocin , released from the posterior pituitary , is the hormone responsible for milk ejection (let-down reflex) during lactation. When a baby suckles, sensory nerves in the nipple send signals to the hypothalamus, triggering oxytocin release . This causes the myoepithelial cells surrounding the alveoli in the mammary glands to contract, forcing milk out through the ducts.
It’s important to distinguish this from prolactin , which stimulates milk production , not ejection.
❌ Why the Other Options Are Incorrect:
Glucagon : Involved in increasing blood glucose; not related to lactation.
None of these : Incorrect—oxytocin is clearly involved.
Prolactin : Stimulates milk synthesis , not milk ejection.
Somatomammotropin (hPL) : Helps with mammary gland development during pregnancy but does not cause milk ejection .
Which week marks the final descent of a gland that began its journey at the base of the tongue and must arrive at its proper location to avoid forming midline neck cysts?
71 / 110
Category:
Endo – Embryology
When does the thyroid gland come in front of the trachea during development?
The thyroid gland begins to develop during the 4th week of embryonic life as a median endodermal thickening in the floor of the primitive pharynx (at the future foramen cecum of the tongue). It descends down the neck through the thyroglossal duct .
By the 7th week , the thyroid gland reaches its final position in front of the trachea , just below the larynx . The thyroglossal duct usually regresses afterward.
This descent is crucial for normal anatomy, and failure of complete descent can result in ectopic thyroid tissue (e.g., lingual thyroid).
❌ Why the Other Options Are Incorrect:
During fetal period : Too vague and non-specific. The thyroid reaches the trachea before the fetal period (which begins at the 9th week).
3rd week : This is before the thyroid even begins to develop.
At birth : Far too late—the gland is fully developed and functional long before birth.
5th month : Again, much too late; the gland has long been in position by this time.
Which hormone on the list is made from cholesterol, travels bound to protein in the blood, and acts directly at the nucleus without needing a second messenger?
72 / 110
Category:
Endo – Biochemistry
Which of the following hormones does not use cyclic adenosine monophosphate (cAMP) as a second messenger?
Most peptide and protein hormones (like FSH, LH, glucagon, and calcitonin) exert their effects via membrane-bound receptors and use second messengers like cyclic AMP (cAMP) to relay the signal inside the cell .
However, estrogen is a steroid hormone . Steroid hormones are lipophilic , so they diffuse through the cell membrane and bind to intracellular (nuclear) receptors , not membrane-bound ones. These hormone-receptor complexes then act directly on DNA , modifying gene transcription—no second messenger like cAMP is involved .
❌ Why the Other Options Are Incorrect:
FSH : Uses cAMP via G-protein-coupled receptors to stimulate follicular development.
Glucagon : Activates adenylyl cyclase → cAMP → activates PKA in liver for glycogen breakdown.
LH : Like FSH, it uses cAMP signaling in gonadal cells.
Calcitonin : Binds to a receptor that stimulates cAMP production to inhibit bone resorption.
What hormone secreted by the pituitary gives you the quickest, most reliable window into how well the thyroid gland is functioning?
73 / 110
Category:
Endo – Pathology
A 54-year-old, married woman para-2 comes to the clinic with the complaint of weight gain and lethargy for the past 3 months. She had her last menstrual period 6 years back. Her blood pressure is 150/90 mmHg, her pulse is 65/min, and is regular bilaterally. Her body mass index (BMI) falls in the obese range. Her eyes are puffy, she complains of breathlessness, and cold intolerance, her skin is coarse and her abdomen has white striations present. What is the best diagnostic test for this woman?
This woman presents with classic signs of hypothyroidism :
Weight gain , lethargy , cold intolerance
Bradycardia (pulse of 65/min), coarse skin , puffy eyes
Menopausal status with metabolic slowing
Hypertension , obesity , and breathlessness (possible due to decreased metabolism or myopathy)
These features strongly point to primary hypothyroidism , and the best initial and most sensitive test to confirm this is the serum TSH level . In primary hypothyroidism, TSH will be elevated , as the pituitary tries to stimulate a failing thyroid.
If the TSH is abnormal, it can be followed by a free T4 test to assess the degree of hormone deficiency.
❌ Why the Other Options Are Incorrect:
Oral glucose tolerance test : Not first-line here. While she may be at risk of diabetes due to obesity, her symptoms are better explained by hypothyroidism.
Serum cortisol : Tests for adrenal insufficiency (e.g., Addison’s), which doesn’t match her presentation.
Serum cortisol and TSH : Unnecessary unless there’s suspicion of multiple endocrine disorders; TSH alone is sufficient initially.
Oral glucose tolerance test and TSH : Again, adds extra testing—TSH alone is the focused and best first test .
When evaluating a syndrome defined by increased cardiovascular and metabolic risk, which specific lipid and glucose-related markers are most critical in its formal diagnosis?
74 / 110
Category:
Endo – Community Medicine/Behavioral Sciences
A 30-year-old man presents to the medical outpatient department with complaints of headache for the past 2 days. Pain is moderate in intensity and associated with blurring of vision. On examination, he has a blood pressure of 160/90 mmHg and body mass index (BMI) of 30 kg/m^2. Which of the following is required to fulfill the definition of metabolic syndrome?
Metabolic syndrome is a cluster of conditions that increase the risk of cardiovascular disease and type 2 diabetes . It is diagnosed when a person has at least three of the following five criteria (as per ATP III / IDF guidelines):
Abdominal obesity (based on waist circumference; BMI can suggest but isn’t directly used)
Elevated triglycerides (≥150 mg/dL)
Reduced HDL cholesterol (<40 mg/dL in men, <50 mg/dL in women)
Elevated blood pressure (≥130/85 mmHg or on antihypertensive medication)
Impaired fasting glucose (≥100 mg/dL or diagnosed diabetes)
The option with impaired fasting glucose, increased triglycerides, and decreased HDL correctly lists three diagnostic components , which is sufficient to define metabolic syndrome .
❌ Why the Other Options Are Incorrect:
Presence of acanthosis nigricans : Suggestive of insulin resistance but not a diagnostic criterion .
History of using corticosteroids : May cause features of metabolic syndrome but is not part of its definition .
Impaired fasting glucose, increased triglycerides, decreased HDL, raised uric acid : Uric acid is not included in the diagnostic criteria.
Raised cholesterol and LDL : Total cholesterol and LDL are not part of the standard diagnostic criteria for metabolic syndrome.
Which hormone released from the pituitary works closely with uterine muscles and the let-down reflex but isn’t involved in making the milk itself?
75 / 110
Category:
Endo – Physio
Hormone secretions from the posterior pituitary include which of the following?
The posterior pituitary (also called the neurohypophysis ) does not synthesize hormones itself—it stores and releases hormones that are produced by the hypothalamus . These hormones are transported down axons from the paraventricular and supraoptic nuclei of the hypothalamus and released into the bloodstream from the posterior pituitary.
The two hormones released by the posterior pituitary are:
❌ Why the Other Options Are Incorrect:
Prolactin : Secreted by the anterior pituitary , stimulates milk production—not ejection.
Thyroxin (T4) : Produced by the thyroid gland , not the pituitary.
Progesterone : Secreted by the corpus luteum in the ovaries, not the pituitary.
Leptin : Secreted by adipose tissue , helps regulate energy balance—not a pituitary hormone.
Which messenger system gets rapidly activated when a hormone needs to act quickly at the cell membrane—particularly one that isn’t lipophilic and can’t enter the cell?
76 / 110
Category:
Endo – Biochemistry
Glucagon is a peptide hormone. The primary target for glucagon is the liver, where its action is mediated by which of the following second messenger systems?
Glucagon is a peptide hormone secreted by pancreatic alpha cells , primarily in response to low blood glucose levels . Its main target is the liver , where it stimulates processes like glycogenolysis and gluconeogenesis to raise blood glucose .
Glucagon exerts its effects through a G-protein-coupled receptor (GPCR) on the hepatocyte membrane. This activates the adenylyl cyclase enzyme, which converts ATP into cyclic AMP (cAMP) —the key second messenger .
cAMP then activates protein kinase A (PKA) , which phosphorylates enzymes that:
Stimulate glycogen breakdown
Promote glucose production
Inhibit glycogen synthesis
❌ Why the Other Options Are Incorrect:
Nuclear receptors : Used by steroid or thyroid hormones , not peptide hormones like glucagon.
cIMP : Not a recognized or physiologically relevant second messenger.
DNA receptors : Hormones don’t act directly on DNA receptors—this is an inaccurate term.
cGMP : Another second messenger, but not the one used by glucagon. It’s used by nitric oxide and atrial natriuretic peptide (ANP), not glucagon.
Which ion normally keeps the cell’s interior negative, and what would happen if the channel for that ion suddenly closed in response to high energy levels?
77 / 110
Category:
Endo – Physio
In the process of release of insulin, beta cells of the pancreas get depolarized by which of the following?
The release of insulin from pancreatic β-cells is a well-regulated process triggered primarily by glucose uptake . Here’s how it works:
Glucose enters the β-cell via GLUT-2 transporters .
It undergoes glycolysis and oxidative phosphorylation , producing ATP .
The rise in ATP leads to the closure of ATP-sensitive K⁺ channels .
This prevents potassium from leaving the cell , causing membrane depolarization .
Depolarization opens voltage-gated Ca²⁺ channels , allowing Ca²⁺ influx .
Increased intracellular Ca²⁺ triggers exocytosis of insulin-containing vesicles.
So, the initial depolarization step is directly caused by ATP-dependent closure of K⁺ channels .
❌ Why the Other Options Are Incorrect:
Entry of Na⁺ by voltage-gated channels : These channels are not central to insulin secretion in β-cells.
ATP-dependent opening of K⁺ channels : It’s closure , not opening, that leads to depolarization.
ATP-dependent closure of Ca²⁺ channels : Ca²⁺ channels are voltage-gated , not ATP-sensitive.
Entry of Ca²⁺ by voltage-gated channels : This happens after depolarization , not before—it triggers insulin release , not the depolarization itself.
What kind of transporter would the thyroid need to bring in negatively charged iodine from the blood into the cell, especially when it’s already present in higher concentration inside?
78 / 110
Category:
Endo – Physio
Which of the following is the transporter responsible for the transport of iodine into the thyroid cells against the concentration gradient?
The transport of iodine into thyroid follicular cells is a critical first step in the synthesis of thyroid hormones (T₃ and T₄). This transport is carried out by the sodium-iodide symporter (NIS) , which moves iodide (I⁻) into the cell against its concentration gradient , using the energy provided by the Na⁺ gradient maintained by the Na⁺/K⁺ ATPase pump.
This is a secondary active transport mechanism:
This process concentrates iodide inside the thyroid cell for organification and hormone synthesis in the follicular lumen.
❌ Why the Other Options Are Incorrect:
I⁻/Cl⁻ antiport : No such transporter is used for iodide uptake in thyroid cells.
K⁺, I⁻ symport : Not involved in iodide transport in thyroid physiology.
Na⁺, glucose symport : Found in the intestine/kidneys for glucose absorption—not related to iodide uptake.
None of these : Incorrect—Na⁺/I⁻ symporter is the correct and well-established mechanism.
If insulin-producing cells sit mostly in the middle of the islet, where would you expect the glucagon-producing cells to be, in order to communicate efficiently but not overlap?
79 / 110
Category:
Endo – Histology
On examining histological slides, the students get confused between the alpha cells and beta cells, forgetting their location in the pancreas. They were informed that the location of alpha cells is in which of the following?
The islets of Langerhans in the pancreas contain several types of hormone-secreting cells, with alpha and beta cells being the most prominent:
Beta cells (which secrete insulin) are usually found in the center of the islets.
Alpha cells (which secrete glucagon) are typically located toward the periphery , though not strictly confined to the outermost edges.
This spatial arrangement helps with paracrine signaling between cell types. For example, insulin from beta cells can inhibit glucagon release from nearby alpha cells, and vice versa.
Histologically, this pattern is more distinct in humans than in some animal models, and it aids in identifying cells under the microscope when using specific immunostains.
❌ Why the Other Options Are Incorrect:
Only in the periphery of islets : Too restrictive—alpha cells are mostly peripheral, but not exclusively.
Equally in both regions of islets : Incorrect—beta cells dominate the center , alpha cells are more peripheral.
Towards the center of islets : That’s where beta cells are located.
Only in the center of islets : Again, incorrect—this applies to beta cells, not alpha cells.
Among the anatomical parts of the pituitary, which one houses the bulk of hormone-producing cells that respond directly to signals from the hypothalamus?
80 / 110
Category:
Endo – Histology
The pituitary gland is a pea-sized gland that plays a major role in regulating vital body functions and general well-being. It is referred to as the body’s master gland because it controls the activity of most other hormone-secreting glands. What part of the anterior pituitary gland is mainly responsible for hormone secretion?
The anterior pituitary , also called the adenohypophysis , is divided into three parts:
Pars anterior (pars distalis) – main site of hormone secretion
Pars intermedia – thin layer with minor role in humans
Pars tuberalis – wraps around the pituitary stalk; limited secretory role
Among these, the pars anterior is the largest and most functionally important region. It contains various hormone-secreting cells that produce:
These hormones are regulated by hypothalamic releasing/inhibiting hormones transported via the hypophyseal portal system .
❌ Why the Other Options Are Incorrect:
Pars intermedia : In humans, this part is mostly rudimentary and not a major source of hormone secretion.
Pars tuberalis : Has some endocrine function but does not contribute significantly to major pituitary hormone output.
Pars nervosa : Part of the posterior pituitary (neurohypophysis), which stores and releases ADH and oxytocin , but does not synthesize hormones .
Hypothalamus : Regulates the pituitary via releasing hormones, but it is not part of the anterior pituitary itself .
Which complication involves damage to major blood vessels and is the primary reason for increased risk of heart attacks and strokes in people with long-standing diabetes?
81 / 110
Category:
Endo – Pathology
What is the marked macrovascular complication of type 2 diabetes mellitus?
In type 2 diabetes mellitus , chronic high blood sugar damages blood vessels throughout the body. Complications can be broadly divided into:
Microvascular (small vessel): Retinopathy, nephropathy, neuropathy
Macrovascular (large vessel): Atherosclerosis , leading to cardiovascular disease
The most prominent macrovascular complication is accelerated atherosclerosis , which increases the risk of:
Coronary artery disease (heart attacks)
Cerebrovascular disease (strokes)
Peripheral artery disease (limb ischemia)
Diabetes promotes inflammation, endothelial dysfunction, and lipid abnormalities , all of which speed up plaque formation in arteries, making cardiovascular disease the leading cause of death in diabetics.
❌ Why the Other Options Are Incorrect:
Retinal vein occlusion : Can occur in diabetes, but it is not the hallmark macrovascular complication ; it is more of a microvascular event .
Tunnel vision : Seen in advanced glaucoma or retinitis pigmentosa , not specific to diabetes.
Diminishing vision : A symptom of diabetic retinopathy , which is a microvascular (not macrovascular) complication.
Accelerated arthritis : Not a recognized complication of type 2 diabetes.
When a patient with an overactive thyroid also presents with eye discomfort and a noticeable change in eye appearance, what underlying immune-related condition should you consider?
82 / 110
Category:
Endo – Pathology
A 30-year-old woman presents to the clinic with palpitations, weight loss, and excessive sweating for almost 5-6 months. She cannot concentrate on carrying out her daily activities especially looking after her children. Recently, she also developed pain in both eyes and her friend told her that one of her eyes has become more prominent than the other. What is the most likely cause of her condition?
This 30-year-old woman presents with classic signs of hyperthyroidism —including palpitations , weight loss , heat intolerance (excessive sweating) , and difficulty concentrating —but what makes this case distinctive is her eye symptoms : pain and proptosis (bulging of the eye) . These features are strongly suggestive of Graves’ disease , which is the only type of hyperthyroidism commonly associated with ophthalmopathy .
Graves’ disease is an autoimmune condition where TSH receptor antibodies (TRAb) stimulate the thyroid gland, causing excess hormone production. These antibodies can also stimulate fibroblasts and immune cells in the orbital tissues, leading to Graves’ orbitopathy (eye pain, swelling, redness, and protrusion).
❌ Why the Other Options Are Incorrect:
Toxic nodular goiter : Causes hyperthyroidism, but not associated with eye symptoms . It’s more common in older individuals and has a more gradual onset.
Generalized anxiety disorder : May cause palpitations and restlessness, but does not explain the weight loss, sweating, or eye findings .
Hyperthyroidism : A general term for excess thyroid hormone—it describes the condition but not the specific cause , which in this case is Graves’.
Hashimoto’s thyroiditis : Typically leads to hypothyroidism , not hyperthyroidism. In early stages, it may transiently cause hyperthyroid symptoms, but does not cause eye changes .
Which hormone becomes active when you’re dehydrated and need to conserve water without necessarily changing your salt levels?
83 / 110
Category:
Endo – Physio
What is the role of anti-diuretic hormone in the human body?
Anti-diuretic hormone (ADH) , also called vasopressin , plays a critical role in maintaining the body’s water balance . It is secreted by the posterior pituitary gland in response to increased plasma osmolality or low blood volume.
ADH acts primarily on the collecting ducts of the kidneys , where it:
Increases water reabsorption by promoting insertion of aquaporin-2 channels into the cell membranes
Concentrates the urine
Reduces water loss, thereby helping maintain plasma volume and osmolality
This hormonal regulation is essential for hydration , blood pressure stability , and overall homeostasis .
❌ Why the Other Options Are Incorrect:
Helps regulate absorption of chloride : Indirectly affected, but not a primary function of ADH.
Helps regulate absorption of phosphate : Mainly controlled by parathyroid hormone (PTH) , not ADH.
Helps regulate absorption of potassium : Regulated by aldosterone , not ADH.
Helps regulate absorption of sodium : Also primarily under the control of aldosterone , not ADH.
Which enzyme would be most efficient in releasing energy quickly from glycogen without producing free glucose, especially during fasting or exercise?
84 / 110
Category:
Endo – Biochemistry
Which of the following enzymes is responsible for cleaving off glucose-1-phosphate from the peripheral end of a long branch of glycogen during glycogenolysis?
During glycogenolysis , the enzyme glycogen phosphorylase is responsible for cleaving glucose residues from the non-reducing (peripheral) ends of glycogen chains. It removes one glucose unit at a time by breaking α-1,4 glycosidic bonds , releasing glucose-1-phosphate .
This is the first and rate-limiting step in glycogen breakdown, and it continues until glycogen phosphorylase reaches about four glucose residues away from a branch point.
❌ Why the Other Options Are Incorrect:
Glucose-6-phosphatase : Converts glucose-6-phosphate to free glucose in the liver , not involved in the initial step of glycogen breakdown.
Branching enzyme : Involved in glycogenesis , not glycogenolysis. It adds α-1,6 branches , not cleaves.
Debranching enzyme : Helps remove branch points after glycogen phosphorylase has done its job, but it doesn’t release glucose-1-phosphate.
Glycogen synthase : Responsible for building glycogen , not breaking it down.
Which artery travels along the superior border of the pancreas on its way to an organ located in the left upper quadrant of the abdomen?
85 / 110
Category:
Endo – Anatomy
Which of the following artery supplies blood to the pancreas?
The splenic artery , a branch of the celiac trunk , is the primary arterial supply to the pancreas , especially to its body and tail . Along its course toward the spleen, the splenic artery gives off pancreatic branches , including:
These arteries form an anastomotic network to ensure robust blood flow to the pancreas.
Other arteries may contribute to the head of the pancreas , such as:
But when you’re asked for the main or direct artery associated with pancreatic blood supply, especially to the body and tail , the answer is the splenic artery .
❌ Why the Other Options Are Incorrect:
Cystic artery : Supplies the gallbladder , not the pancreas.
Inferior esophageal artery : Supplies the lower esophagus ; no direct role in pancreatic blood supply.
Left gastric artery : Supplies the stomach and lower esophagus , not the pancreas.
Inferior mesenteric artery : Supplies the hindgut (e.g., distal colon and rectum); it does not reach the pancreas .
If the body lacks the hormone that normally tells the kidneys to hold onto salt and dump potassium, what would happen to blood salt and potassium levels?
86 / 110
Category:
Endo – Physio
Which of the following will be seen in mineralocorticoid deficiency?
Mineralocorticoids like aldosterone play a key role in regulating electrolytes by acting on the distal nephron in the kidneys. Their main functions are:
Increase sodium reabsorption
Increase potassium excretion
Promote water retention (indirectly via sodium)
So, in mineralocorticoid deficiency (e.g., Addison’s disease), the opposite happens:
Sodium is lost in the urine , leading to hyponatremia and low blood pressure
Potassium is retained , leading to hyperkalemia
Water is lost , leading to hypovolemia
❌ Why the Other Options Are Incorrect:
Decrease reabsorption of Na and K : K⁺ is actually retained , not lost—so this is incorrect.
Decrease reabsorption of Ca and hypokalemia : Calcium isn’t primarily regulated by mineralocorticoids; hypokalemia is opposite of what actually happens.
None of them : Incorrect—one of the options is correct.
Increase reabsorption of Na : That would happen with excess aldosterone, not deficiency.
Which layer of the adrenal cortex would be most likely to support secondary sex characteristics, rather than controlling blood pressure or glucose?
87 / 110
Category:
Endo – Histology
Which of the following secretes androgens?
The adrenal cortex has three distinct layers , each responsible for producing different hormones. These can be remembered using the mnemonic:GFR = Salt, Sugar, Sex
Zona Glomerulosa → Mineralocorticoids (like aldosterone – regulates salt)
Zona Fasciculata → Glucocorticoids (like cortisol – regulates sugar)
Zona Reticularis → Androgens (like DHEA – sex hormones)
Therefore, androgens are mainly secreted by the zona reticularis of the adrenal cortex.
❌ Why the Other Options Are Incorrect:
Adrenal medulla : Secretes catecholamines (epinephrine and norepinephrine), not androgens .
Zona fasciculata : Primarily produces cortisol , not androgens.
Zona glomerulosa : Produces aldosterone , not androgens.
All of these : Incorrect—only zona reticularis among the listed options produces androgens.
Consider which electrolyte plays a crucial role in muscle cell depolarization and is actively lost in excess due to the hormone involved in Conn syndrome.
88 / 110
Category:
Endo – Physio
Which of the following causes lead to muscle weakness in Conn syndrome?
Conn syndrome , or primary hyperaldosteronism , is caused by excess production of aldosterone from the adrenal glands. Aldosterone promotes sodium retention and potassium excretion by the kidneys. Over time, this leads to hypokalemia —low potassium levels in the blood.
Potassium is essential for normal muscle function , especially for maintaining proper electrical activity in muscle cells. When potassium levels drop, muscles become less excitable , leading to muscle weakness, cramps, and fatigue .
❌ Why the Other Options Are Incorrect:
Hypertension : Common in Conn syndrome but does not directly cause muscle weakness.
Hypernatremia : Rare in Conn syndrome because the body compensates by increasing water retention; also not a major cause of muscle weakness.
Hypercalcemia : Not a feature of Conn syndrome. While it can cause muscle weakness, it isn’t the relevant mechanism here.
Neuropathy : Not typically part of Conn syndrome and not the main reason for muscle weakness in this condition.
If infections are well controlled and rare, what other internal process could quietly destroy the glands without any external threat?
89 / 110
Category:
Endo – Pathology
What is the most common cause of Addison’s disease in the West?
In Western countries, the most common cause of Addison’s disease is autoimmune adrenalitis . This means the body’s immune system attacks and destroys its own adrenal glands, leading to a shortage of important hormones like cortisol and aldosterone. It often occurs with other autoimmune conditions like type 1 diabetes or thyroid problems.
❌ Why the Other Options Are Wrong:
Tumor of adrenal gland : More likely to cause extra hormone production, not hormone loss.
Congenital absence of adrenal gland : Very rare, and mostly affects newborns—not adults.
Infection : Can damage the adrenal glands, but it’s not the top cause in the West.
Tuberculosis : A major cause in developing countries, but much less common in the West today.
If you were looking at a gland under a microscope and saw a faint cell in the middle of an acinus (instead of ducts with stripes), which organ might you be looking at?
90 / 110
Category:
Endo – Histology
What differentiates the pancreas from the parotid gland?
Both the pancreas and the parotid gland are exocrine glands with similar-looking structures, such as serous acini (clusters of enzyme-secreting cells). However, key histological differences help distinguish one from the other under a microscope.
The most distinguishing feature of the pancreas is the presence of centroacinar cells —these are pale-staining cells found in the center of the acini , and they represent the beginning of the intercalated duct system . You do not see these in the parotid gland.
Additionally, the pancreas lacks striated ducts , which are typically seen in salivary glands like the parotid. Striated ducts are responsible for ion transport and appear with eosinophilic stripes due to infoldings of the basal membrane packed with mitochondria.
So, the combination of:
Presence of centroacinar cells , and
Absence of striated ducts is what makes the pancreas histologically distinct.
❌ Why the Other Options Are Incorrect:
Compound glands are present in the pancreas : ❌ True, but not unique —the parotid gland is also a compound gland , so this doesn’t help differentiate.
Presence of striated ducts in the pancreas : ❌ Incorrect. Striated ducts are absent in the pancreas. They are found in parotid and other salivary glands .
Serous acini are present in the parotid, not in the pancreas : ❌ False. Both glands have serous acini , although their secretions differ in composition (digestive enzymes in pancreas, salivary enzymes in parotid).
Secretory cells arranged in a circle around a central duct in the pancreas : ❌ This is a vague and nonspecific description, and doesn’t clearly separate it from the parotid gland histologically.
Think about a messaging system where a surface signal sets off a chain reaction inside the cell using a “cyclic” middleman—what’s the most common system for fast-acting peptide hormones?
91 / 110
Category:
Endo – Physio
Parathyroid hormone, growth hormone-releasing hormone, and glucagon all use which signaling pathway?
Parathyroid hormone (PTH) , growth hormone-releasing hormone (GHRH) , and glucagon are all peptide hormones that cannot cross the cell membrane. So instead of acting inside the cell, they bind to cell surface receptors and use second messengers to deliver their signal inside.
The most common second messenger for these hormones is cyclic AMP (cAMP) . Here’s how it works:
The hormone binds to a G-protein coupled receptor (GPCR) on the cell surface.
This activates the Gs protein , which then stimulates the enzyme adenylate cyclase .
Adenylate cyclase converts ATP into cAMP .
cAMP activates protein kinase A (PKA) , which then triggers a cascade of effects inside the cell.
This cAMP signaling pathway is used by:
PTH : to increase calcium reabsorption and activate vitamin D
Glucagon : to increase blood glucose through glycogen breakdown
GHRH : to stimulate release of growth hormone from the anterior pituitary
❌ Why the Other Options Are Incorrect:
cGMP : ❌ Used by very few hormones (like nitric oxide and ANP), not PTH, glucagon, or GHRH.
Intracellular receptor : ❌ Used by steroid and thyroid hormones , which are lipid-soluble and enter the cell directly—not peptide hormones like these.
Receptor tyrosine kinase (RTK) : ❌ Used by insulin and growth factors like IGF, but not by glucagon, PTH, or GHRH.
IP3 (Inositol triphosphate) : ❌ Used by hormones like ADH (via V1 receptors) and oxytocin —not by the ones listed here.
Think about which step in the pathway adds a chemical “tag” to norepinephrine, making it more potent during stress—and which enzyme makes that final modification possible.
92 / 110
Category:
Endo – Biochemistry
Which of the following enzymes is needed to convert norepinephrine to epinephrine?
The synthesis of catecholamines (dopamine, norepinephrine, epinephrine) follows a stepwise enzymatic pathway , primarily taking place in the adrenal medulla and certain neurons.
Here’s the simplified pathway:
Tyrosine → (via Tyrosine hydroxylase ) →
DOPA → (via DOPA decarboxylase ) →
Dopamine → (via Dopamine β-hydroxylase ) →
Norepinephrine → (via Phenylethanolamine N-methyltransferase , or PNMT ) →
Epinephrine
The final step , converting norepinephrine to epinephrine , is catalyzed by PNMT . This enzyme adds a methyl group to norepinephrine, forming epinephrine. Importantly, PNMT activity is stimulated by cortisol , which comes from the nearby adrenal cortex—showing how the adrenal medulla and cortex function in coordination .
❌ Why the Other Options Are Incorrect:
Monoamine oxidase (MAO) : ❌ Breaks down catecholamines; it’s involved in degradation , not synthesis.
DOPA hydroxylase : ❌ There’s no such enzyme. The correct early enzyme is tyrosine hydroxylase , which forms DOPA.
Catechol-O-methyltransferase (COMT) : ❌ Also involved in breaking down catecholamines, not synthesizing them.
Tyrosine hydroxylase : ❌ This enzyme acts at the start of the pathway—converting tyrosine to DOPA , not norepinephrine to epinephrine.
Think of the islet like a bullseye: which cell type, responsible for reducing blood sugar, sits at the center of this endocrine target?
93 / 110
Category:
Endo – Histology
Which of the following types of cells are majorly placed centrally in the pancreas?
The pancreas has both exocrine and endocrine functions. The endocrine part is made up of clusters of cells called the Islets of Langerhans , which are scattered throughout the pancreas and contain several types of hormone-producing cells.
Among these:
Beta (β) cells :
Produce insulin
Make up about 60–70% of the islet cells
Are located centrally in the islets
Insulin helps lower blood glucose
Alpha (α) cells :
Delta (δ) cells :
PP cells (F cells) :
So, the beta cells , which produce insulin, are the ones primarily located in the center of each islet.
❌ Why the Other Options Are Incorrect:
PP cells : ❌ Found in smaller numbers and in more specific regions—not centrally located in the islets.
Alpha cells : ❌ Found on the outer rim (periphery) of the islets.
Delta cells : ❌ Scattered and not dominant in number; not centrally concentrated.
None of these : ❌ Incorrect—beta cells are clearly centrally located in the islets.
Think of the tail of the pancreas as stretching its arm toward the spleen—what deep organ lies just behind it on that side of the body?
94 / 110
Category:
Endo – Anatomy
The pancreatic tail is present anterior to which structure?
The pancreas lies retroperitoneally (behind the peritoneal cavity), and it’s divided into:
Head : Nestled in the curve of the duodenum
Neck and body : Cross the midline behind the stomach
Tail : Extends toward the left side , ending near the hilum of the spleen
Now, the tail of the pancreas passes anterior to the left kidney as it travels through the splenorenal ligament to reach the spleen. This is an important anatomical relationship because:
In surgery (e.g., splenectomy), the tail of the pancreas is at risk of injury
It lies anterior to the upper pole of the left kidney and the left suprarenal gland
❌ Why the Other Options Are Incorrect:
Liver : ❌ Located in the right upper quadrant , far from the tail of the pancreas, which is on the left .
Sigmoid colon : ❌ Found in the lower left abdomen , below the level of the pancreatic tail.
Esophagus : ❌ Lies superior and posterior to the stomach and does not relate directly to the pancreas in this region.
Right kidney : ❌ Located on the right side , related more to the head and uncinate process of the pancreas—not the tail , which is on the left .
When your body needs to hold on to water, which hormone signals the kidneys to stop sending it all out in urine? The absence of this signal causes the tap to run nonstop.
95 / 110
Category:
Endo – Physio
Diabetes insipidus is caused by deficiency of which hormone?
Diabetes insipidus (DI) is a condition where the body loses too much water through urine, leading to excessive thirst (polydipsia) and excessive urination (polyuria) . But unlike diabetes mellitus , there is no high blood sugar .
The problem in diabetes insipidus is a deficiency or resistance to ADH (antidiuretic hormone) , also known as vasopressin .
ADH is produced in the hypothalamus and released by the posterior pituitary gland . Its main job is to conserve water by acting on the kidneys to reabsorb water from the urine back into the blood.
If there’s not enough ADH (or the kidneys don’t respond to it), the result is:
Types of DI:
❌ Why the Other Options Are Incorrect:
Insulin : ❌ Controls blood sugar , not water balance. Its deficiency causes diabetes mellitus , not DI.
Aldosterone : ❌ Regulates sodium and potassium , and indirectly water—but its deficiency causes Addison’s disease , not DI.
Angiotensin II : ❌ Helps raise blood pressure and stimulate aldosterone—not the main hormone in DI.
FSH (Follicle Stimulating Hormone) : ❌ Involved in reproductive hormone control , not water regulation.
When the body prepares for action under stress, certain organs get activated to perform more, while others are suppressed—but not all tissues need a direct nerve signal to respond. Which one is more about structure and less about instant reaction?
96 / 110
Category:
Endo – Physio
Adrenergic receptors are present in all of the following except:
Adrenergic receptors are specialized proteins located on the surface of cells that respond to catecholamines like epinephrine (adrenaline) and norepinephrine . These receptors are part of the sympathetic nervous system , which controls the “fight or flight” response.
There are several types of adrenergic receptors (α1, α2, β1, β2, β3), and they are found in many tissues throughout the body.
Here’s a quick overview of where they are found and what they do:
Heart : ✅ Contains β1 receptors → Increase heart rate and contractility.
Intestines : ✅ Contain mostly α and β receptors → Slow down digestion during stress.
Skeletal muscle : ✅ Contains β2 receptors → Cause vasodilation to increase blood flow.
Viscera (internal organs like liver, kidneys, etc.): ✅ Adrenergic receptors help regulate blood flow, metabolism, and smooth muscle tone.
However:
Bone : ❌ Bones do not have a significant number of adrenergic receptors for catecholamines. Bone metabolism is mainly regulated by hormones like parathyroid hormone (PTH), vitamin D, calcitonin, and sex hormones —not through adrenergic signaling. While some studies suggest possible indirect effects via sympathetic tone, bone is not a major adrenergic target .
❌ Why the Other Options Are Incorrect:
Heart : ✅ β1 receptors are heavily involved in sympathetic stimulation of the heart.
Intestines : ✅ Sympathetic activation slows down motility via α2 receptors.
Skeletal muscle : ✅ β2 receptors cause vasodilation to supply more oxygen and glucose.
Viscera : ✅ Adrenergic receptors modulate blood flow and smooth muscle in organs like kidneys, liver, bladder, etc.
Sometimes in development, structures that start higher end up lower—especially when they hitch a ride with migrating neighbors. Which gland descends with the thymus?
97 / 110
Category:
Endo – Embryology
The inferior parathyroid glands develop from which of the following?
The parathyroid glands develop from the pharyngeal pouches , which are embryological structures found in the developing head and neck region.
There are four parathyroid glands :
Two superior
Two inferior
Here’s the key point:
Even though the 3rd pouch lies above the 4th during development, the inferior parathyroids end up lower because they descend with the thymus , which also develops from the ventral part of the 3rd pouch .
So:
❌ Why the Other Options Are Incorrect:
Dorsal 2nd pharyngeal pouch : ❌ Forms palatine tonsils—not related to the parathyroid glands.
Dorsal 4th pharyngeal pouch : ❌ Gives rise to the superior parathyroid glands, not inferior.
Ventral 4th pharyngeal pouch : ❌ Develops into the ultimobranchial body , which becomes C-cells (parafollicular cells) of the thyroid—not parathyroids.
Ventral 3rd pharyngeal pouch : ❌ Gives rise to the thymus , not the parathyroids.
Within the thyroid gland, some cells focus on metabolism, while others act more quietly to help regulate bones and calcium—often found lurking between the main players.
98 / 110
Category:
Endo – Histology
Which of the following cells produce calcitonin?
Calcitonin is a hormone involved in calcium homeostasis . It helps lower blood calcium levels by:
Calcitonin is produced by the parafollicular cells (also called C cells ) of the thyroid gland . These cells are found in between the follicles of the thyroid (hence “para”-follicular) and are not involved in thyroid hormone production (which is the job of follicular cells).
These C cells come from a neural crest origin , specifically from the ultimobranchial body , which fuses with the thyroid during development.
❌ Why the Other Options Are Incorrect:
Chief cells : ❌ Found in the parathyroid gland , and they produce parathyroid hormone (PTH) , which increases blood calcium—not calcitonin.
Islets of Langerhans : ❌ These are in the pancreas , and they produce insulin, glucagon, somatostatin , etc.—unrelated to calcitonin.
Chromaffin cells : ❌ Found in the adrenal medulla , these produce catecholamines (epinephrine and norepinephrine), not calcitonin.
Thyroid follicular cells : ❌ These produce thyroxine (T4) and triiodothyronine (T3) , not calcitonin.
If the brain’s “thermostat” is trying to cool things down, but the system keeps heating up anyway, where do you think the problem originates—the controller or the machinery?
99 / 110
Category:
Endo – Pathology
A 35-year-old female went to see her doctor because she had been feeling hot lately and was sweating at night. Her T3 and T4 levels were found to be high, whereas her TSH level was low. What is the most likely diagnosis made by the doctor?
This patient shows classic signs of hyperthyroidism —feeling hot, night sweats, and likely other symptoms like weight loss, palpitations, or anxiety. Her lab results reveal:
This pattern tells us that the thyroid gland itself is overactive , producing too much hormone. The pituitary gland senses this excess and reduces TSH production to try to bring hormone levels down—this is a normal negative feedback response .
This is called primary hyperthyroidism , because the problem starts in the thyroid gland itself .
❌ Why the Other Options Are Incorrect:
Hypothyroidism : ❌ In this condition, T3 and T4 would be low , not high.
Myxedema : ❌ This is a severe form of hypothyroidism , often with very low thyroid hormone levels and high TSH —not consistent with the patient’s findings.
Pituitary adenoma : ❌ A tumor in the pituitary that secretes TSH would cause high TSH along with high T3 and T4—this patient has low TSH , so this isn’t the cause.
Secondary hyperthyroidism : ❌ This means the pituitary gland is causing the problem , typically with elevated TSH , which then stimulates the thyroid. But here, TSH is low , ruling this out.
Think about a structure that starts as part of the mouth, reaches up toward the brain, and eventually loses its original connection. When during development do such changes typically complete?
100 / 110
Category:
Endo – Embryology
Regarding the development of pituitary gland, which of the following is true?
The pituitary gland (hypophysis) has a dual embryological origin , which is why understanding its development is a bit more complex.
It forms from two parts:
Rathke’s pouch – an ectodermal outpouching from the roof of the stomodeum (primitive mouth). This gives rise to the anterior pituitary (adenohypophysis), including:
Pars distalis
Pars intermedia
Pars tuberalis
Infundibulum – a neuroectodermal outgrowth from the floor of the diencephalon (part of the brain). This forms the posterior pituitary (neurohypophysis), including:
Pars nervosa
Infundibular stalk
As development progresses:
Rathke’s pouch grows upward toward the brain.
The infundibulum grows downward .
The connection between Rathke’s pouch and the stomodeum disappears by the 12th week , leaving a closed structure.
❌ Why the Other Options Are Incorrect:
The entire gland develops from Rathke’s pouch : ❌ Incorrect. Only the anterior pituitary develops from Rathke’s pouch. The posterior part comes from neuroectoderm .
It is purely ectodermal in origin : ❌ Incorrect. The anterior pituitary is ectodermal (from oral ectoderm), but the posterior part comes from neuroectoderm , so it’s not purely ectodermal.
Hypophyseal diverticulum arises from the floor of stomodeum : ❌ Trick statement. The roof of the stomodeum gives rise to Rathke’s pouch , not the floor .
Pars nervosa develops from Rathke’s pouch : ❌ Incorrect. Pars nervosa (part of posterior pituitary) comes from the infundibulum (neuroectoderm) , not Rathke’s pouch.
If you wanted to stop a factory from producing a product, would you cut off the raw materials, block the assembly line, or destroy the warehouse? Consider what step this drug targets in hormone production.
101 / 110
Category:
Endo – Pharmacology
Which of the following is the effect of propylthiouracil on thyroid hormone?
Propylthiouracil (PTU) is a medication used to treat hyperthyroidism , especially in conditions like Graves’ disease . It works by directly reducing the production of thyroid hormones (T3 and T4) in the thyroid gland.
PTU inhibits two key steps in thyroid hormone synthesis:
Oxidation of iodide to iodine – This is the step where iodide (I⁻) is converted to its active form (I₂), which is essential before it can be added to the tyrosine residues on thyroglobulin.
Coupling of iodotyrosine residues – This is the step where MIT (monoiodotyrosine) and DIT (diiodotyrosine) combine to form T3 and T4 .
Both of these steps are catalyzed by the enzyme thyroid peroxidase (TPO) , and PTU blocks this enzyme.
Additionally, PTU also inhibits peripheral conversion of T4 to T3 , which is important because T3 is the more active form .
❌ Why the Other Options Are Incorrect:
Inhibit T3 release from thyroglobulin : ❌ PTU doesn’t affect the release of already-formed hormones; it blocks their formation , not their secretion .
Hyper activates Na-I symporter : ❌ PTU does not increase iodine uptake; that’s the job of TSH . The symporter brings iodide into the cell, but PTU acts after that.
Inactivates thyroglobulin : ❌ PTU doesn’t destroy thyroglobulin. Thyroglobulin is the storage protein for T3 and T4 inside the thyroid follicles.
Hypertrophy of thyroid follicles : ❌ That’s a result of excess TSH stimulation , often seen in iodine deficiency or long-term PTU use due to negative feedback , but it’s not a direct effect of PTU.
Imagine you are in a clinic without a calculator, scale, or imaging machine. What simple tool could help you predict someone’s risk of heart disease and diabetes just by checking where fat is stored?
102 / 110
Category:
Endo – Community Medicine/Behavioral Sciences
What is the simplest parameter to define obesity in metabolic syndrome?
In the context of metabolic syndrome , the most practical and simplest way to define obesity is by measuring waist circumference .
Metabolic syndrome is a group of risk factors—including high blood pressure, high blood sugar, abnormal cholesterol, and central (abdominal) obesity —that together increase the risk of heart disease and diabetes.
Why waist circumference? Because it specifically reflects abdominal (visceral) fat , which is more closely linked to metabolic complications than overall body fat.
It’s:
Simple (just use a tape measure)
Quick (no calculations)
Effective (strong predictor of disease risk)
Recommended by guidelines (e.g., ATP III, IDF)
Thresholds:
❌ Why the Other Options Are Incorrect:
Body weight : ❌ Doesn’t distinguish between fat and muscle or show where fat is located.
Body mass index (BMI) : ❌ Requires calculation and may miss people with normal weight but high belly fat (common in South Asians). It doesn’t specifically measure central obesity.
Measurement of visceral fat by MRI scan : ❌ Very accurate but not practical —expensive, time-consuming, and not used in routine screenings.
Waist-hip ratio : ❌ Useful, but more complex than waist circumference and less direct in predicting visceral fat .
Consider how a fat-soluble molecule might behave differently from water-soluble ones—how deep into the cell could it go to influence change, and where would its signal be permanently recorded?
103 / 110
Category:
Endo – Physio
Steroid hormones bind to which of the following structures?
Steroid hormones (like cortisol, estrogen, testosterone, and aldosterone) are lipid-soluble , which means they can easily pass through cell membranes .
Once inside the cell, they do not act on the surface like most peptide hormones. Instead, they enter the nucleus , bind to specific intracellular receptors , and then this hormone-receptor complex binds to a special region on the DNA called the Hormone Response Element (HRE) .
Binding to the HRE allows the complex to directly influence gene expression —either turning genes on or off—leading to changes in protein synthesis. This is how steroid hormones have long-term effects , such as growth, metabolism changes, or reproductive functions.
❌ Why the Other Options Are Incorrect:
None of these : ❌ Incorrect—there is a correct structure: the hormone response element.
G-protein coupled receptors (GPCRs) : ❌ These are surface receptors used by peptide hormones (like adrenaline, glucagon), not steroids .
Second messengers : ❌ These are intracellular signaling molecules (like cAMP or IP3) used when hormones bind outside the cell —not used by steroid hormones , which act inside the nucleus.
Enzyme-linked receptors : ❌ These are also cell surface receptors , such as tyrosine kinase receptors used by insulin—not relevant to steroid hormone action.
Think about which pituitary cells would go into overdrive if the body suddenly started growing too much—who’s in charge of growth?
104 / 110
Category:
Endo – Physio
If the growth hormone level in the blood is increased due to a tumor of the pituitary gland, then which cells of the pituitary gland are most likely affected by the tumor?
The pituitary gland is like the master control room for hormones in the body. It contains different types of cells, and each type makes a specific hormone.
Somatotropes are the cells that produce growth hormone (GH) . If there is a tumor in the pituitary and GH levels are high, it means that somatotropes are overactive or have grown into a tumor —called a somatotroph adenoma .
This leads to excessive GH in the blood, causing conditions like:
Gigantism (in children, before bones stop growing)
Acromegaly (in adults, after growth plates have closed)
❌ Why the Other Options Are Incorrect:
Thyrotropes : These cells make TSH (thyroid-stimulating hormone) , which controls the thyroid gland—not related to GH.
Lactotropes : These produce prolactin , the hormone involved in milk production. Prolactinomas are common, but they cause high prolactin—not GH.
Gonadotropes : These produce LH and FSH , which affect the ovaries and testes—not GH.
Corticotropes : These make ACTH , which stimulates the adrenal glands—not GH.
Imagine you’re looking at the brain from below—like the brain is upside down in front of you. Which parts would be clearly sticking out, and which ones are hidden inside the deeper layers?
105 / 110
Category:
Endo – Anatomy
Which of the following is not observed in the inferior part of the hypothalamus when it is viewed from below?
When you look at the underside of the brain (the bottom view), you can see several parts of the hypothalamus .
Things that you can see from below include:
Mammillary bodies – two small round bumps.
Infundibulum – the stalk that connects the brain to the pituitary gland.
Tuber cinereum – a small area of gray matter near the center.
Optic chiasma – the “X-shaped” area where the optic nerves cross.
But the tegmentum is different. It’s not part of the visible underside of the hypothalamus. Instead, it lies deeper inside the brain , behind the hypothalamus, as part of the brainstem (especially the midbrain).
So, if you’re looking at the bottom of the brain, you won’t see the tegmentum —it’s hidden.
❌ Why the Others Are Visible:
Mammillary bodies – ✅ yes, small bumps under the hypothalamus
Infundibulum – ✅ yes, the pituitary stalk
Tuber cinereum – ✅ yes, in the middle part of the hypothalamus
Optic chiasma – ✅ yes, in front of the hypothalamus
Tegmentum – ❌ no, it’s deeper and part of the brainstem, not on the surface
If you’re standing in front of a mirror with just a tape measure, which simple tool could give you a meaningful insight into your risk for metabolic disease—without needing a calculator or lab report?
106 / 110
Category:
Endo – Community Medicine/Behavioral Sciences
Which of the following is an accurate and simple parameter, requiring no calculation, to assess central obesity?
Central obesity means fat is mainly stored around the abdomen , which is more dangerous than fat in other parts of the body because it increases the risk of heart disease, diabetes, and other metabolic problems.
The simplest and most direct way to assess central obesity—without needing any calculations —is by measuring waist circumference . It only requires a tape measure , and it’s done by measuring around the abdomen at the level of the belly button (umbilicus) .
Health risks increase when:
This method is:
❌ Why the Other Options Are Incorrect:
Weight measurement : Only tells you total body weight. It doesn’t show where fat is located (e.g., arms vs. belly).
Blood cholesterol levels : These are lab-based risk markers , not measurements of body fat or obesity.
Body Mass Index (BMI) : Requires a calculation using both weight and height. It also doesn’t distinguish between muscle and fat or tell where fat is distributed.
Weight-to-height ratio : Also requires calculation , and like BMI, it doesn’t focus on abdominal fat specifically.
Think of a hormone that acts like a referee between two opposing players on the blood sugar team, signaling both to pause rather than letting one dominate.
107 / 110
Category:
Endo – Physio
Which of the following is decreased by somatostatin?
Somatostatin is like the “brake” of the endocrine system. It is a hormone released by the delta (δ) cells of the pancreas and also by parts of the brain and gut. Its job is to slow down or stop the secretion of many other hormones.
In the pancreas , somatostatin mainly inhibits the release of both insulin and glucagon .
This dual inhibition helps fine-tune blood sugar control and prevents sudden spikes or drops.
❌ Why the Other Options Are Incorrect:
Insulin : Somatostatin does decrease insulin, but not only insulin—it also decreases glucagon, so this option is incomplete.
Cortisol : This is produced by the adrenal cortex in response to ACTH from the pituitary. Somatostatin has no direct effect on cortisol secretion.
Glucagon : Again, somatostatin does decrease glucagon, but it also decreases insulin—so this option is also incomplete .
Aldosterone : Another hormone from the adrenal cortex, involved in salt and water balance. Somatostatin does not significantly affect aldosterone levels.
Think about the vessel responsible for delivering hormones from the hypothalamus to the anterior pituitary—where that connection happens might guide you toward the right artery.
108 / 110
Category:
Endo – Anatomy
Which artery supplies the infundibulum?
The infundibulum is the stalk that connects the hypothalamus to the pituitary gland —specifically, to the posterior lobe of the pituitary. It’s a crucial part of the brain’s hormonal control system.
The superior hypophyseal artery , a small branch of the internal carotid artery , is the main vessel that supplies blood to the infundibulum , the upper part of the pituitary gland, and the median eminence of the hypothalamus.
These arteries form a network of capillaries called the primary capillary plexus , which is part of the hypothalamo-hypophyseal portal system —a system that carries releasing hormones from the hypothalamus down to the anterior pituitary.
❌ Why the Other Options Are Incorrect:
Basilar artery : Supplies the brainstem and cerebellum. It does not send branches to the pituitary or infundibulum.
Internal carotid artery : It’s the main source of blood to the brain and gives rise to the superior hypophyseal artery , but it does not directly supply the infundibulum.
External carotid artery : Supplies the face, scalp, and superficial head structures , not the brain or pituitary.
Inferior hypophyseal artery : This artery supplies the posterior lobe of the pituitary, but not the infundibulum itself.
Think about the part of the brain that would “nudge” you to get up and find food or water when you’re running low—rather than just adjusting hormone levels quietly behind the scenes.
109 / 110
Category:
Endo – Physio
Which nucleus of the hypothalamus is involved in the regulation of thirst and hunger?
The hypothalamus is like the brain’s control center for basic needs—things like eating, drinking, sleeping, and staying at the right temperature.
One important part of the hypothalamus is the lateral nucleus , which helps start hunger and thirst . When your body needs food or water, this area tells you to eat or drink. If this area gets damaged, a person might stop feeling hungry or thirsty.
Scientists found this out by studying animals: when they damaged the lateral nucleus, the animals wouldn’t eat or drink even when they were starving or dehydrated.
❌ Why the Other Options Are Wrong (Made Easy):
Paraventricular nucleus : This area helps you feel full and also controls stress hormones and water balance. It’s more about stopping eating than starting it.
Ventromedial nucleus : This is called the satiety center —it tells you when you’re full. If it’s damaged, you might eat too much and gain weight.
Supraoptic nucleus : This part makes a hormone called vasopressin (ADH) that helps your body keep the right amount of water, but it doesn’t make you feel thirsty.
Suprachiasmatic nucleus : This is your body’s internal clock . It helps control your sleep-wake cycle and daily rhythms, not hunger or thirst.
When your body is low on fuel or water, which part sends out the first signal to go find some ? It’s not the one that stops you, but the one that gets you going.
110 / 110
Category:
Endo – Physio
Which nucleus of the hypothalamus is involved in the regulation of thirst and hunger?
The hypothalamus helps control many body functions, including hunger and thirst. Among its many parts, the lateral nucleus is especially important because it starts the feeling of hunger and thirst . When this part is active, it makes you want to eat or drink.
If this area gets damaged, a person may stop eating and drinking , leading to weight loss and dehydration.
Other choices are not mainly responsible for starting hunger or thirst:
Paraventricular nucleus helps with hormones like oxytocin and CRH, more related to stress and water balance.
Ventromedial nucleus tells the body when to stop eating. If it’s damaged, people eat too much.
Supraoptic nucleus makes ADH, which helps the kidneys hold onto water. It doesn’t directly cause thirst.
Suprachiasmatic nucleus controls your sleep and daily rhythm, not hunger.
Your score is
The average score is 0%
Follow us on our Socials ! Thank you.
Restart quiz
Anonymous feedback
Thank you for your feedback.